Das Ende des Physikalismus? Patrick Spät über den graduellen Panpsychismus

BLOG: Natur des Glaubens

Evolutionsgeschichte der Religion(en)
Natur des Glaubens

Mal wieder eine nette Blog-Überraschung: Zum 300ten Blogpost von “Natur des Glaubens” kam u.a. ein Gratulationsmail des Philosophen Patrick Spät, dessen Der Mensch lebt nicht vom Hirn allein” ich vor einiger Zeit hier rezensiert hatte und das auch in anderen Medien rege Resonanz gefunden hat.

DerMenschlebtnichtvomHirnalleinSpaet

Dieses Online-Kontaktchance ließ ich mir natürlich nicht entgehen und bat Patrick Spät um ein Web-Interview – und darum, Fragenden und Diskutierenden hier auch zehn Tage zur Verfügung zu stehen. Und er sagte zu!

Hier ist also das Web-Interview mit Patrick Spät zum Thema Panpsychismus vs. Physikalismus.

1. Dr. Spät, mir begegnen vor allem in der Blogosphäre immer wieder “Physikalisten”. Sie behaupten, wir alle seien nur eine Ansammlung von Atomen, vertreten diese Auffassung aber mit emotionaler Erregung und großem, missionarischen Eifer. Und sie scheinen auch sehr motiviert zu sein, Geist und Bewußtsein Anderer zu beeinflussen. Haben Sie eine Erklärung für dieses sehr widersprüchliche Verhalten? Woher bekommen Atomhaufen Gefühle, Motive, Geist, Bewusstsein und Lust am Argumentieren?

Das liegt wohl an der übertriebenen Wissenschaftsgläubigkeit, die wir seit drei Jahrhunderten erleben: Die Naturwissenschaft scheint die Allmacht zu haben, den Kosmos, das Leben und schließlich den Menschen zu entzaubern, also all das auf mathematisch-physikalische Modelle zu reduzieren und damit buchstäblich erklärbar zu machen. Warum es dann aber überhaupt denkende Wesen gibt, die Fragen stellen und eine Weltsicht haben können, kann uns der Physikalist nicht klarmachen.

Natürlich haben naturwissenschaftliche Erkenntnisse ihre Gültigkeit, doch ihr Gültigkeitsbereich ist eben beschränkt – und zwar auf mathematische Gleichungen. Solche grauen Formeln sind abstrakt, während das bunte Leben konkret ist: Wenn ich mich wie im Film “Die Fliege” verkleinere und in Ihrem Gehirn umherfliege, dann sehe ich elektro-chemische Signale. Aber ich “sehe” nicht, wie es sich für Sie anfühlt, wenn Sie gerade den leckersten Wein Ihres Lebens trinken – das können nur Sie selbst erleben. Und es ist absurd, dass es jemals ein Buch geben wird, das das Gefühl der Liebe voll und ganz erklärt. Wir sind keine Waschmaschinen, deren Funktionen wir lückenlos in einer Anleitung beschreiben könnten.

Der Panpsychismus versucht, sowohl den körperlichen – also naturwissenschaftlich vermessbaren – Aspekten wie auch den geistigen gerecht zu werden: Es gibt körperlich-materielle und es gibt geistige Eigenschaften, die bereits in der Wurzel der Wirklichkeit stets zusammen auftreten. Schon ganz weit unten, bei den Atomen etwa, gibt es simple geistige Eigenschaften. Das heißt aber nicht, dass ein Atom Schmerzen hat, wenn ich Kernspaltung betreibe. Erst dann, wenn ein Ding in materieller Hinsicht ausreichend komplex ist, nimmt auch seine geistige Seite komplexere Formen an. Deshalb haben nur Menschen und einige Tiere wie etwa Schimpansen, Elstern oder Elefanten ein Selbstbewusstsein ihrer selbst. Ich glaube, dass es in der Natur eine graduelle Ordnung gibt, die das Ergebnis der Evolution ist. Geist und Körper bilden eine untrennbare Symbiose, die immer komplexere Formen annimmt.

2. Was kann der Panpsychismus noch, was der Physikalismus nicht kann?

Zuerst verschafft der Panpsychismus dem Geistigen den Platz, der ihm gebührt: Das Geistige ist ein Teil der Welt; kein Mysterium, sondern ein natürliches Phänomen wie Ladung, Masse und Spin auch. Allerdings berücksichtigt der Panpsychismus, dass sich das Geistige deutlich vom Körperlichen abgrenzt: Das konkrete Wummern und Pochen, das Sie spüren, wenn Sie sich in den Finger schneiden, lässt sich nicht durch Formeln einfangen. Der Geist ist Geist – nicht mehr und nicht weniger.

Ein weiteres Problem des Physikalisten ist, dass er behauptet, alles, wirklich alles sei rein physikalischer Natur. Dabei kann er aber nicht mal im Ansatz erklären, was Materie überhaupt ist: Die Naturwissenschaft zeigen, wie sich Materie verhält, aber nicht, was sie ihrem Wesen nach wirklich ist. Wenn Ihnen der Chemiker sagt, Zucker sei C12H22O11, dann ist das korrekt, aber eben nur die halbe Miete: Denn der Geschmack der Süße gehört auch zu dieser Welt, da ihr erlebendes Bewusstsein zweifelsohne Teil der Welt ist.


Dr. Patrick Spät, dessen Bucherfolg “Der Mensch lebt nicht vom Hirn allein” u.a. von Harald Lesch empfohlen wurde.

3. Gibt es wirkliche Unterschiede zwischen einem graduellen Panpsychismus und dem Emergentismus?

Auf jeden Fall! Der Physikalist – und der Emergentismus ist nur eine Spielart dieser Weltsicht – muss das Geistige hundertprozentig aus der Welt verbannen. Für ihn gibt es am Fundament der Welt nur nackte Materie, die wir mit unseren Formeln lückenlos beschreiben können. Der Emergentist glaubt, dass aus dem rein materiellen Hirn plötzlich geistige Eigenschaften “auftauchen”. Damit glaubt er natürlich, dass wir aus Stroh Gold gewinnen können: Das Universum soll aus reiner Materie bestehen, dann kommen die Lebewesen und mir nichts dir nichts macht es “plopp” und der Geist ist da. Wie dieser Hokuspokus möglich sein soll, kann der Emergentist nicht erklären. Der graduelle Panpsychismus hingegen behauptet, dass der Körper eines Lebewesens als “Verstärker” wirkt und die simplen geistigen Eigenschaften – die im Fundament der Wirklichkeit verwurzelt sind – bündelt und intensiviert. Falls es hier eine Form der Emergenz gibt, dann ist sie sehr schwach. Und vor allem ist nachvollziehbar, woher der Baum des Bewusstseins kommt: aus dem Samen des fundamentalen Geistigen. Wie allerdings diese simplen geistigen Eigenschaften konkret aussehen, kann der graduelle Panpsychismus nicht ganz klären – mit Atomen können wir ja keinen Kaffeeplausch halten.

4. Auf Twitter kursiert ein schöner Witz: “Ein Notfall – Wir brauchen einen Philosophen!” Wie reagieren Sie?

Spontan schießt mir ein englisches Sprichwort in den Kopf: “When all is said and done, more is said than done”. Philosophen versuchen, die Welt zu deuten. Und eine gute Theorie ist hier und da die beste Praxis. Aber unsere Welt ist voll von akuten Notfällen, die wir mit bloßem Theoriegeplauder nicht lösen werden. Die Ironie des Twitter-Spruchs halte ich also für ziemlich berechtigt. Auch angesichts der vielen Alibi-Philosophen, die in Talkshows eingeladen werden, dort aber oft sehr zaghaft unterwegs sind, statt den Finger in die Wunde zu legen. Theorien, und dazu gehört klarerweise auch der Panpsychismus, können vielleicht an den Rändern des kollektiven Bewusstsein ihre Spuren hinterlassen. Damit wäre schon viel gewonnen, weil ich glaube, dass die Ausbeutung des Menschen und des Planeten einen Gutteil ihrer Wurzeln in der physikalistischen Weltsicht haben.

Vielen Dank für dieses Web-Interview, Dr. Spät – und dafür, dass Sie den Leserinnen und Lesern von Natur des Glaubens noch zehn Tage für die Online-Debatte zur Verfügung stehen! Möge Ihr Buch unter manchem Weihnachtsbaum zu finden sein!

* Diesen Blogpost widme ich auch dem Philosophen Stephan Schleim, dessen Brainlog mich seinerzeit angeregt hatte, auch wieder verstärkt philosophische Bücher in den Lesestapel aufzunehmen und der inzwischen auch auf scilogs.com (Englisch) gegen den Physikalismus anbloggt.

Avatar-Foto

Dr. Michael Blume studierte Religions- und Politikwissenschaft & promovierte über Religion in der Hirn- und Evolutionsforschung. Uni-Dozent, Wissenschaftsblogger & christlich-islamischer Familienvater, Buchautor, u.a. "Islam in der Krise" (2017), "Warum der Antisemitismus uns alle bedroht" (2019) u.v.m. Hat auch in Krisenregionen manches erlebt und überlebt, seit 2018 Beauftragter der Landesregierung BW gg. Antisemitismus. Auf "Natur des Glaubens" bloggt er seit vielen Jahren als „teilnehmender Beobachter“ für Wissenschaft und Demokratie, gegen Verschwörungsmythen und Wasserkrise.

121 Kommentare

  1. Dualismus

    Vielen Dank für das aufschlussreiche Interview. Ich wundere mich ja immer, dass Menschen den Ansatz des Panpsychismus für eine tiefere Erklärung halten als Emergenz. Im einen Fall sagen wir, das Bewusstsein entsteht durch komplexen Zusammenspiel von Neuronen. Im anderen Fall sagen wir, das geistliche war halt schon immer irgendwie da. Klar müssen wir dann nicht mehr erklären, wie es entsteht. Aber verstanden haben wir es damit immer noch nicht.

    Dr. Spät freut sich darüber, dass die Physikalisten nicht beweisen können, das Bewusstsein aus körperlichen Vorgängen entsteht. Er selbst kann aber seine dualistische Auffassung auch nicht beweisen. Oder ist das geistliche eines Atomkerns irgendwie messbar?

  2. @Joachim: Au ja! 🙂

    Nach dem Higgs-Boson für die Masse nun eines für den Geist – das wäre ja dann mal wirklich fast ein “Gottesteilchen”! 🙂

    Aber ich bin gespannt, wie Spät selbst antwortet und daher jetzt still.

  3. Was ist wirklich?

    “Emergenz” ist eher ein Zauberwort als eine Erklärung. Daher spricht die amerikanische Biologin Ursula Goodenough etwa auch vom “Wunder der Emergenz”.
    Warum muss eigentlich immer alles durch Reduktionismus erklärt werden: entweder Geist durch Materie oder Materie durch Geist? Sind das nicht Zwangshaltungen veralteter Metaphysiken (Materialismus, Idealismus)? Warum ist es nicht möglich, sowohl die Wirklichkeit des Materiellen als die des Seelisch-Geistigen anzuerkennen und sorgfältig zu erforschen? Dafür haben wir doch Natur- und Geisteswissenschaften, Psychologie, Literatur, Kunst. Es kann eine genauso sorgfältige und präzise Erforschung der inneren Wirklichkeiten geben, wie eine der äusseren. Wir leben in beiden. Ob sie nur Seiten EINER Medaille sind, ist noch nicht ausreichend zu beantworten. Daher vielleicht etwas mehr Bescheidenheit, als zwanghafte Kausalerklärungen in der einen oder anderen Richtung.

  4. Physikalisten glauben in der Regel eben nicht, dass mit den vorhandenen Theorien alles erklärbar ist. Aber wenn sie etwas erklären, ist es belegbar, objektivierbar und auch nachvollziehbar. Wenn sie spekulieren ist es auch klar, dass sie spekulieren und spekulieren tut fast jeder, weil es einfach Spaß macht. Dass sich die Panpsychiker allein aufs Spekulieren beschränken macht sie nicht attraktiver.

  5. Schon ganz weit unten, bei den Atomen etwa, gibt es simple geistige Eigenschaften.

    Kann man die auch sehen, schmecken, hören oder fühlen? Oder existieren diese Eigenschaften, weil jemand sagt, dass es sie gibt?

  6. Ich bin “Emergentist”, wenn man das so verkürzt überhaupt sagen kann, und halte den Physikalismus für falsch. – Und aus diesem Grund halte ich das, was Spät da über das Verhältnis von Physikalismus und Emergentismus sagt, für groben Unsinn.

    Der Physikalismus behauptet, dass man jegliche beobachtbare Erscheinung auf die Wechselwirkung von Atomen reduzieren kann. Der Emergentismus hingegen, dass das *prinzipiell* nicht möglich ist. Wie man diese beiden unvereinbaren Theorien in einen Topf werfen kann, ist mir völlig schleierhaft.

    Was aber jegliche Weltanschauung und jegliche Wissenschaft leisten muss, ist, dass sich die einzelnen Beschreibungen nicht im Widerspruch zueinander befinden.

  7. Ich bin ja bisher eher unbeleckt von so Diskussionen um Physikalismus und Emergenz, und das Wort „Panpsychismus“ lese ich heute zum ersten Mal. Aber überzeugt hat mich dieses Interview nicht gerade.
    Ich behaupte mal, dass Reduktionisten (= Physikalisten?) keineswegs blind für Gefühle, Bewusstsein usw. sind, sondern sich im Gegenteil sehr dafür und für deren Ursachen und Herkunft interessieren. Ob das zu wissen nützlich ist, weiß ich nicht, hochspannend finde ich es jedenfalls. Und wo soll man sonst nach Ursachen für alles suchen, wenn nicht im Aufbau von allem? Im „Samen des fundamentalen Geistigen“ vielleicht? Verzeihung, aber das klingt nach alles und nichts erklärender Esoterik und „nachvollziehbar“ finde ich das überhaupt nicht.
    Ich bin sicher, dass es extrem kompliziert ist komplexes Verhalten durch einfaches zu erklären. So kompliziert, dass man „Zwischenschichten“ einführen muss, in denen man mithilfe der Gesetze aus den unteren Schichten komplexere Gesetze formuliert (Diese Zwischenschichten gibt es auch schon, sie nennen sich Psychologie, Biologie, Hirnforschung, Chemie, Physik, Mathematik). So kompliziert, dass wahrscheinlich nie ein Mensch existieren wird, der die Intelligenz dazu hat, diese Zwischenschichten zu überspringen und den Menschen „auszurechnen“. Aber so kompliziert, dass es prinzipiell unmöglich ist? Na na na, ich glaube nicht. Ich finde, man sollte generell vorsichtig sein mit Sätzen wie „Man wird nie X erklären können“.

    Aber vielleicht geht mein Kommentar hier auch, siehe erster Satz, ganz an der Diskussion vorbei 🙂

  8. Suggestivfragen eines Gleichdenkers

    Die Haltung “Was ihr wollt” geb ich euch, schafft Dinge wie den Panpsychismus und die Interviewfrage
    “Was kann der Panpsychismus noch, was der Physikalismus nicht kann?”
    passt zu dieser Wünsch-Dir-Was Haltung.

    Warum aber sollte gerade das Phänomen des Psychischen/Geistigen so etwas Herausragendes und vor allem Elementares sein, dass es eine eigene Physik des Geistigen braucht.

    Andere Phänomene als das Psychisch/Geistige, von denen wir heute wissen, dass sie nicht elementar sind, haben früher zu ganz ähnlichen “Materialisationen” in Form eigener Kräfte/Prinzipien geführt, beispielsweise die Vier-Elemente-Lehre

    Die Eigenschaften einer Flüssigkeit wie Wasser und alles was mit Wasser und Flüssigkeit zusammenhängt schafft bereits eine Realität, die sich mindestens intuitiv nicht auf grundlegende physikalische Prinzipien zurückführen lässt. Doch heute wissen wir, dass flüssig-sein auf grundlegende Physik rückführbar ist.

    Was bedeutet denn ein physikalistisches oder materialistisches Weltbild für unser Leben? Wohl einfach, dass es eine Basis von allem gibt und diese Basis materiell ist. Wir wissen aber auch, dass diese Basis für das meiste was unser Leben ausmacht uninteressant ist. Was uns bewegt, motiviert, antreibt, all das existiert auf einer sehr viel höheren Ebene als es die Ebene der physikalischen Prinzipien ist.

    Es ist sogar anzunehmen, dass es mehrere Realisationsmöglichkeiten für die gleichen komplexen Dinge gibt, die unser Leben bestimmen. Genau so wie es verschiedene Transportmittel gibt, gibt es wahrscheinlich auch verschiedene Arten wie man ein intelligentes, motiviertes, Emotionen verspührendes Geschöpf kreiren kann. Wir Menschen sind nur eine solche Materialisation. Es gibt weitere Möglichkeiten dies zu tun und einige von unserer Gattung werden Geschöpfe mit Geist und Psyche, mit Intelligenz, Wille und der Frage nach dem Sinn von allem selber mittels Software und Hardware kreiren – und das noch in absehbarer Zeit.
    Und einige dieser Geschöpfe werden sicher an der Idee des Panpsychismus Gefallen finden.

  9. Antworten

    Liebe Kommentatorinnen und Kommentatoren,

    herzlichen Dank für Ihre Beiträge und Fragen!

    @ Joachim:
    Ja, es ist in der Tat schwierig bis unmöglich, den Panpsychismus zu “beweisen”. Und vollends verstehen werden wir das Geistige vermutlich nie. Aber wir erleben es, was m.E. “beweist”, dass es “da ist”. Und unsere Erlebnisse zeigen, dass das Geistige Eigenschaften hat, die das Materielle (so wie es Physikalisten definieren) nicht hat. Deshalb glaube ich, dass der Panpsychismus der Emergenztheorie einen entscheidenden Schritt voraus ist. Denn “aus Nichts kommt Nichts” …

    @ Rüdiger Sünner:
    Vollste Zustimmung! Beide Seiten gehören zur Wirklichkeit – und eine Theorie, die die gesamte Wirklichkeit beschreiben möchte, muss m.E. all diese Seiten berücksichtigen.

    @ adenosine:
    Hmm, klar spekuliere ich: Der Naturwissenschaftler misst und wiegt, der Philosoph interpretiert das ganze – und spekuliert dabei. In der Welt des Physikalismus, der den Naturwissenschaften eine “alles-erklärende” Position zuspricht, gibt es aber keine Deutungen der Welt. Anders formuliert: Ein konsequenter Physikalist glaubt, dass alles um uns herum mit Formeln und Gleichungen erklärbar ist, zumindest im Prinzip. Auch wenn die Quantentheorie oder das Bewusstsein harte Nüsse zu sein scheinen, gehen Physikalisten dennoch davon aus, dass all diese Dinge/Eigenschaften letztlich auf materiellen Prozessen basieren.

    @ Martin B. und Martin Holzherr:
    Gute Frage! “Beweisen” kann man wohl nichts, was da unten im Kellergeschoss der Wirklichkeit vor sich geht. Da schiebt uns schon die Planck-Skala einen Riegel vor. Der Panpsychismus ist eine Theorie – und ich denke, dass sich diese Theorie aus einigen Argumenten ergibt, deren Gegenannahmen zu kusiose(er)n Theorien führt.

    @ Köppnick:
    Entweder geht der Emergentist davon aus, dass alles materiell ist: Dann hat er den Salat, den ich im Interview versucht habe zu beschreiben. Oder er geht davon aus, dass das Geistige “irgendwie” in der Wirklichkeit vorhanden ist. Dann kann er Panpsychist, Idealist, Dualist oder anders sein. Ich sehe keinen (logischen) Ausweg aus diesem Dilemma.

    @ Daniel:
    Hmm, viele Philosophen gehen davon aus (zu recht, wie ich glaube), dass das Geistige tatsächlich einzigartige Eigenschaften hat: selbstreflexive Gedanken, Intentionalität, qualitative Eigenschaften (z.B. beim Erleben unserer fünf Sinne) usw.
    Flüssig-sein ist AUCH (nicht nur) ein geistiges Phänomen: Es braucht ein erlebendes Wesen, dass etwas Flüssiges sieht, hört, fühlt. Ähnliches gilt für Farben.
    Ansonsten stimme ich zu!

    @ Jens:
    Mich auch 🙂

    @ alle:
    Der graduelle Panpsychismus ist natürlich ein theoretischer Vorschlag zur Aufdröselung der Leib-Seele-Problems, keine wasserdichte Naturgesetzlichkeit, die ich im Labor “beweisen” kann.
    Danke für die Kommentare und beste Grüße
    P. Spät

  10. @Holzherr, @Köppnick & Spät

    Ich möchte gerne an drei Diskutanten je zu den Begriffen “Materie” und “Emergenz” nachfragen.

    Bei “Materie” ist mir nicht klar, warum diese tot bzw. geistlos sein sollte. Kultur- und geistesgeschichtlich ist doch gerade auffällig, dass der Urstoff schon in der Antike (und wahrscheinlich noch sehr viel früher) als Mater-ia als “Mutterstoff” bezeichnet wurde! Die dualistische Trennung von Stoff und Leben bzw. Geist ist m.E. doch erst sehr viel später geschehen und hat sich m.E. als empirisch sehr fragwürdige Spekulation erwiesen.
    https://scilogs.spektrum.de/…s-geheimnis-des-urstoffes

    Und zu Emergenz: Ich kann nicht erkennen, wie sich Emergentismus und Panpsychismus notwendig ausschließen sollten. Die Emergentistin erkennt doch m.E. zunächst einfach nur an, dass neuartige (System-)Eigenschaften im Zeitverlauf beobachtbar auftreten. M.E. ist damit doch noch gar keine Letztaussage verbunden, ob diese schon immer (“panpsychisch”) in der grundlegenden Materie “verwurzelt” oder durch zufällig-evolutionäre Prozesse, durch göttliches Eingreifen o.ä. hinzugefügt werden. Mir scheint es ein Vorzug des Emergentismus zu sein, dass er an dieser Stelle bis zum Auftauchen neuer Erkenntnis aus guten Gründen stehenbleiben und agnostisch sein kann. Oder habe ich da etwas völlig falsch verstanden?

  11. Panpsychismus,Kreationismus,Welterklärun

    Kreationismus/Evolution erklären auf unterschiedliche Art wie Automaten/Organismen, Intelligenz und Zweckgerichtetheit in die Welt kommen: nämlich entweder durch einen Schöpfer oder aber durch eine Kombination von Zufall und Selektion.

    Panpsychismus/Emergenz erklären auf unterschiedliche Art wie Bewusstsein in die Welt kommt: entweder durch die Präexistenz/Koexistenz von Bewusstsein mit Materiellem oder aber durch Organisation des Materiellen.

    Warum beschäftigt das Körper-Geist-Problem sogar mehr Philosopen als es das Problem der Existenz von sich selbst erhaltenden Organismen tut?
    Wohl weil gilt: “Ich denke, also bin ich”? oder sogar (für einen Philosophen) “ich äussere mich und werde wahrgenommen, deshalb bin ich”.

    Wer sich der Welt und seiner selbst über Denken versichern “muss”, der will auch Gewissheiten wo es keine gibt und er wird äusserst unzufrieden, ja geradezu grantig, wenn er folgende Situation vorfindet: “Denn der Materialist kann uns nicht sagen, was ein Hormon ist, was ein Neuron ist und was schließlich Materie ist. Über das Wesen der Dinge muss die Naturwissenschaft schweigen: sie kann nur mit Formeln arbeiten.”

    Absolute Gewissheit erreicht man mittels Beweisen, doch “Exakte, nicht anzweifelbare Definitionen sind einzig in der Mathematik und in Teilen der Naturwissenschaften möglich. Dass man ‘Bewusstsein’ nicht exakt definieren kann, deutet schon an, dass dieser Begriff Eigenschaften herausgreift, die sich in der naturwissenschaftlichen Sprache weder beschreiben noch erklären lassen.”

    Ontologische Aussagen im Sinne von “das ist das Wesen eines x”, wobei ein x ein Elektron, die Liebe oder ein Hirn ist , kann die Naturwissenschaft tatsächlich nicht machen. Statt dessen macht sie Aussagen über Beziehungen zwischen Dingen und akzeptiert die Dinge und ihre Beziehungen so wie sie sind, denn (Feynman)“Nobody knows how it can be like that”
    Anders als viele Geisteswissenschaftler meinen, kann die Physik als Grundlage aller anderen Naturwissenschaften kaum etwas beweisen und will es auch gar nicht oder wie Richard Feynman sagt:
    “Scientific knowledge is a body of statements of varying degrees of certainty — some most unsure, some nearly sure, but none absolutely certain.”

    Wer von der Physik her die Welt erklären will, für den ist die Welt die Summe aller miteiander interagierenden materiellen Dinge und soweit er die Interaktionen versteht soweit versteht der Physiker die Welt. Gibt es aber ein Phänomen, das man nicht versteht erfindet der Naturwissenschaftler nicht ohne Not etwas Neues um die Lücke zu füllen, sondern er versucht zuerst einmal die Lücke mit dem schon existierenden Wissen abzudecken.
    Um unsere Alltagswelt zu erklären, haben die Physiker bis jetzt keine Lücke im physikalischen Grundverständnis entdeckt. Vieles was uns Menschen beschäftigt wird jedoch von der Physik nicht “motiviert”. Warum ein Mensch das aber nicht etwas anderes tun will oder warum Schach ein interessantes Spiel ist wird von der Physik in der Tat nie erklärt werden. Und trotzdem wird kein Schachzug, weder derjenige, der in Gedanken noch derjenige, der mit einer menschlichen Hand ausgeführt wird, gegen die bekannten physikalischen Gesetze verstossen.

    Wer mehr wissen will als man wissen kann, der erfindet sich halt seine Welterklärung und er findet da draussen sicher ein paar Buddies, die sich ihm mit Freude und Erleichterung anschliessen.

  12. Eigene Meinung als Tatsachen ?

    “Schon ganz weit unten, bei den Atomen etwa, gibt es simple geistige Eigenschaften.”

    Welche Eigenschaften sind es? Ganz konkret bitte. Durch welche Experimente wurde dies festgestellt?

    “Erst dann, wenn ein Ding in materieller Hinsicht ausreichend komplex ist, nimmt auch seine geistige Seite komplexere Formen an. Deshalb haben nur Menschen und einige Tiere wie etwa Schimpansen, Elstern oder Elefanten ein Selbstbewusstsein ihrer selbst.”

    Welche Belege gibt für diese Behauptungen?
    Was heißt hier “komplex”?
    Sind Kühe weniger komplexer als Elster und deswegen “dümmer”?

  13. Eintopf

    Eintopf ist ein leckeres Gericht; es hilft aber nichts, wenn Begriffe wie Materie, Leben/Tod, Geist undifferenziert zusammengewürfelt werden.
    Ein wesentlicher Unterschied zwischen belebter und unbelebter Natur/Materie ist die Materieeigenschaft ´Lernfähigkeit´. So kann z.B. unbelebte Materie (Steine) nicht lernen, auf Umwelteinflüsse aktiv zu reagieren. Pflanzen, Tiere und Menschen sind dagegen in der Lage einfache Reflexreaktionen auszuführen (z.B. Pflanzen) oder aber erlerntes Wissen zu erwerben, zu speichern, um dann auf einen Reiz auf unterschiedliche Weise zu reagieren (Tiere, Menschen).

  14. Das ist doch…

    nichts anderes als wissenschaftliche Unbildung seitens dieses Philosophens. Es gibt auch tatsächlich Philosophen die das wissenschaftliche Weltbild verstanden haben…

    Gleichwohl bildet Spät einen Großteil des “Zeitungs”-status-quo ab.

  15. @Blume: Wie misst man PsycheImAtom?

    Das folgende ist eine falsche Frage: Bei “Materie” ist mir nicht klar, warum diese tot bzw. geistlos sein sollte

    Die richtige Frage ist dagegen: Wie interagiert Geist mit Materie? Wie lässt sich das Wirken von Geist messen?

    Was nicht interagiert, existiert nicht. Wir könnten uns eine Parallelwelt genau vor unserer Nase vorstellen, wenn diese Parallelwelt aber nichts mit uns austauscht und auch nichts mit uns austauschen kann, existiert sie für uns nicht.

    Was miteinander interagiert kann unserer Erfahrung nach auf physikalische Interaktionen zurückgeführt werden. Von ihren Gedanken erfahren andere nur, wenn sie sie aussprechen oder anderswie äussern. Egal wie sie das tun, es sind physikalische Interaktionen und messbare (physikalische) Energieumwandlungen bei diesem Vorgang beteiligt.

    Bei der materialistischen Sicht beginnen wir mit Dingen, die für alle unzweifelhaft existieren und deren Beobachtung und Manipulation für alle Beobachter unabhängig von ihrer Identitiät vergleichbare Resultate bringt. So etwas wie Geist gehört nicht zu diesem initialen Set von Dingen, mit dem wir starten, denn es ist offensichtlich, dass Geist etwas Komplexes ist, welches unserer Erfahrung nach an materielle Dinge wie Personen gebunden ist, wo wir aber aufgrund der Komplexität nicht oder noch nicht wissen, was es ausmacht.

    Für den Materialisten/Physiker ist Geist zuerst einmal etwas ähnliches wie ein UFO, also ein unbekanntes Flugobjekt. Man könnte Geist/Psyche als UAO bezeichnen, also als Unkown Acting Object bezeichnen. Ein UFO und ein UAO ist aber Untersuchungen zugänglich, auch physikalischen Untersuchungen.

  16. @Spät

    Ihre Behauptungen zeugen von Unverständnis des momentanen wissenschaftlichen Standpunktes. Sie leben von Strohmännern, die so weit vom tatsächlichen holistischen Standpunkt entfernt sind, dass ich bezweifle, dass sich hier einer der naturwissenschaftlichen Blogger bereit erklärt überhaupt eine Antwort zu schreiben. Geschweige denn eine Diskussion mit Ihnen zu führen.
    Ihr Standpunkt ist einfach nur zum fremdschämen….

  17. @Martin Holzherr

    Das physikalistische Sprachspiel überzeugt mich nicht: Hier argumentieren ja eine ganze Menge Leute, eindeutig geistig (und teilweise erkennbar emotional) motiviert durch die Suche nach Wahrheit, nach Argumenten, Versuche der Herabsetzung oder Überzeugung Andersdenkender, Bewunderung oder Vorbehalten gegenüber Philosophen etc. Und diese Menschen bewegen zum Beispiel aufgrund ihrer geistig-emotionalen Motive ihre Finger, um Tasten zu drücken, veranlassen damit komplexe elektronische Mechanismen, die wiederum Symbole auf unserem Bildschirmen erscheinen lassen, die wir anderen dann wiederum geistig (!) entschlüsseln – kurz: Wir alle hier bewirken also durch Geistiges auch beobachtbar Physikalisches, das wiederum auf unser Geistiges rückwirkt.

    Gerade wenn wir den religiösen oder wissenschaftlichen Dualismus hinter uns lassen wollen (der m.E. schon durch Läsionsanalysen deutlich geschwächt ist), muss das Geistige also schon immer im Physischen verwurzelt oder später (wie?) hinzugetreten sein. Beobachtbar existent und wirksam ist es ja auf jeden Fall – auch hier auf diesem Blog.

    Wenn Sie also schreiben: So etwas wie Geist gehört nicht zu diesem initialen Set von Dingen, mit dem wir starten, denn es ist offensichtlich, dass Geist etwas Komplexes ist, welches unserer Erfahrung nach an materielle Dinge wie Personen gebunden ist, wo wir aber aufgrund der Komplexität nicht oder noch nicht wissen, was es ausmacht., dann frage ich mich schon, wer da startet (“…mit dem wir starten…”) und wer da sieht (“…ist offensichtlich…”) – Sie setzen das Geistige damit auch hier ja bereits wieder voraus!

    Auch würde ich doch gerne die “einfachere” Frage stellen, wo Geistiges herkommt, bevor wir diskutieren, “was es ausmacht”. Und mich darüber wundern, wie viel darüber offenkundig nicht gewusst und dennoch mit missionarischer Inbrunst verkündet wird…

  18. @Anton Maier

    Ich stand kurz davor, Ihren unsachlichen Kommentar zu löschen, habe mich aber aus Respekt vor früheren Debatten für das Antworten entschieden.

    Nur zwei Hinweise:
    1. Sie argumentieren leider gerade nicht, Sie dogmatisieren und greifen persönlich an. Einer vermeintlich rational-wissenschaftlichen Weltanschauung entspricht das sicher nicht.

    2. Mein Gast Dr. Spät hat seinen Doktortitel nicht geschenkt bekommen, sondern bei renommierten Lehrenden erworben. Und sein Buch wird u.a. von Prof. Harald Lesch ausdrücklich empfohlen, dem durchaus naturwissenschaftliche Kompetenz zuzutrauen ist.

  19. @Michael Blume: Sind wir aus TonGeformt?

    Die Frage: Auch würde ich doch gerne die “einfachere” Frage stellen, wo Geistiges herkommt, bevor wir diskutieren, “was es ausmacht”.

    Vor dieser Frage steht die Frage woher all die Geschöpfe und der Mensch herkommt. Eine Frage, die die Menschen schon lange beschäftigt hat und die auch die meisten Religionen meinten beantworten zu müssen. Interessanterweise sind viele Erklärungen auch in Büchern wie dem alten Testament materialistisch geprägt.
    Der Mensch wird dort aus Ton geformt.

    Das einzige nicht-materialistische daran ist der Schöpfer, der den Ton formt. Allerdings scheint das nicht einmal den Menschen dazumal genügt zu haben. Deshalb wurde dem Menschen noch der Lebensatem eingehaucht.

    Materialistische Welterklärungen sind übrigens uralt und wurden lediglich von den alten Griechen und von Denkern der Neuzeit wiederentdeckt.

    Wenn man nicht bereit ist, Komplexeres mittels Einfacherem/Grundsätzlicherem zu erklären – wie wie das offenbar sind -, dann steht man letztlich vor sehr vielen Rätseln. Müsste man dann nicht sogar Dinge wie Wolken als nichtreduzibel und eigenständige Objekte betrachten und sich Fragen stellen wie

    Auch würde ich doch gerne die “einfachere” Frage stellen, wo Wolken herkommen, bevor wir diskutieren, “was Wolken ausmachen”.

    Das Geistige und unsere Denkfähigkeit erscheint uns sicher darum als etwas Besonderes, weil wir damit die Welt erkennen können und damit das verstehen lernen was uns hervorgebracht hat. Wir sind bis jetzt das einzige bekannte Produkt dieser Welt dass diese Welt sogar verstehen kann.

  20. @Blume

    Sie haben offensichtlich meinen Kommentar nicht gelesen.
    zu1: wenn ich sage, dass Späts Schlussfolgerungen auf Strohmännern beruhen ist das keine dogmatisierung. die strohmänner sind so krass, dass man sich nur fremdschämen kann… vielleicht empfindet spät das als beleidigung, aber das ist halt einfach der realität ins auge gesehen. hier auf die argumentation späts einzugehen wäre in etwa so als wollte man mit junge-erde-kreationisten diskutieren. die werfen auch mit strohmännern um sich dass man bis zum hals in stroh steht und genau aus diesem grund habe ich überhaupt keine lust mich durch diesen berg stroh durchzuwühlen. eine möglichkeit wäre eine “frage-antwort”-diskussion, weil nur eine art bisektion der argumente in der lage wäre sich durch den berg stroh durchzuwühlen. diskussionen a la oxford-debate-club sind anfällig für stroh-kanonen. strohwerfen ist quasi eine dns-attacke auf oxford-debates. dabei unterstelle ich spät nichtmal böse absicht, sondern schlicht und einfach unbildung

    zu2: das argument ist in der tat unsachlich und ein roter hering für die ich nicht noch mehr zeit verschwenden werde genauso wie ich mich nicht mit strohwerfern auseinandersetze.

  21. @blume nachtrag

    nachtrag zu1:
    auch eine antwort als blogbeitrag ist durch strohmänner dns gefährdet und genau deswegen bezweifel ich, dass sich überhaupt jemand die mühe macht. bisektion wäre aus meiner sicht die einzige möglichkeit den arbeitsaufwand zu bewältigen.

  22. @Spät @Blume

    Daniel Dennett hat sein Leben dem Mind-Thema gewidmet und erklärt den wissenschaftlichen Holismus für Humanisten, Philosophen und andere Geisteswissenschaftler in der gebührenden strenge, die Geisteswissenschaftler an den Tag legen. Für die die es Ernst meinen gibt es einige Bücher von Dennett. Das Problem ist, dass die Spekulationen von Emergenz und Dualismus so viel Stroh produzieren, dass ein Buch nicht reicht…

  23. “Geist der Atome”

    “Schon ganz weit unten, bei den Atomen, gibt es simple geistige Eigenschaften.”
    Das klingt natürlich waghalsig und hypothetisch, aber vielleicht kann mir ein kompetenter Naturwissenschaftler hier auf dem Blog mal folgende einfache Frage beantworten:
    Warum verspürten vor Jahrmilliarden Elementarteilchen das Bedürfnis, sich mit anderen zu Atomen zu verbinden? Warum taten diese sich zu Molekülen zusammen, diese zu Zellen etc.?
    Zufall, Überlebensmaximierung, Intelligenz? Zumindest doch eine starke Aktivität oder?

  24. Liebe Kommentatoren,

    hui, viele Fragen …

    @ Emergenz-Fragende …
    Ich glaube, dass spätestens Descartes das getrennt hat, was zusammenehört, namentlich Geist und Materie. Die “Entgeistigung” und völlige Mathematisierung der Materie mag für die konkrete Alltagspraxis im Labor und am Computer hilfreich sein – für unser Weltbild ist sie katastrophal.

    Die Emergenz von neuen Eigenschaften ist m.E. etwas tricky: Wenn aus H2 und O etwas Flüssiges entsteht (H2O, also Wasser), dann ist hierbei jeder Schritt naturwissenschaftlich nachvollziehbar. Und auf qualitativer Ebene entsteht nichts völlig neuartiges, was beim geistigen anders gelagert ist:
    Hier müsste aus Materie – wenn sie denn passiv und geistlos ist – etwas hervorgehen, dass sich fundamental von der verursachenden Ebene unterscheidet. Die mathematisierbare Materie denkt nicht und fühlt nicht. Wie kommen Träume, Gedanken, Emotionen, Wünsche etc. in die Welt? Zumindest die Spuren solcher Eigenschaften und Dinge müssen in der Wurzel der Wirklichkeit verankert sein – sie können nicht vom Himmel gefallen sein.
    Im Gegenzug glaube ich, dass die plötzliche und völlige Emergenz geistiger Eigenschaften IHRERSEITS unserem naturwissenschaftlichen Weltbild zuwiderläuft:

    Die Emergenztheorie ist eine Spielart des Physikalismus. Deswegen muss sie die physikalistische These akzeptieren, dass sich alle Phänomene nach unabänderlichen und berechenbaren Naturgesetzen richten. Dann kann aber der Geist nicht aus der Materie auftauchen. Denn die Emergenztheorie geht ja davon aus, dass sich die emergenten Eigenschaften nicht aus den Eigenschaften der verursachenden Ebene erklären lassen: Nichts aus dieser Ebene gibt auch nur einen Hinweis darauf, wie das emergente Produkt beschaffen ist. Demzufolge wären die grundlegenden Gesetze der Physik, Biologie und Chemie
    lückenhaft, da sie das Auftauchen geistiger Eigenschaften nicht erklären können. Diese Lückenhaftigkeit läuft wiederum einem naturwissenschaftlich orientierten Physikalismus zuwider, den die Emergenztheorie aufrechtzuerhalten versucht: Man kann nicht zugleich einen strikten Physikalismus psotulieren UND und die Existenz qualitativer Sprünge annehmen. Etwas rein Materielles kann einzig und allein etwas rein Materielles bedingen und hervorrufen! Aus Wasser bekommen wir auch nur Wasser, aber niemals Wein. Ein mysteriöses Auftauchen des Geistigen steht
    m.E. im Widerspruch zu allen uns bekannten Naturgesetzlichkeiten und philosophischen Überlegungen. Das menschliche Denken kapituliert, will es sich das Hervorgehen von etwas Seiendem aus dem Nichtseienden verständlich machen.

  25. @ Maxim und Was-ist-da-unten-Fragende:

    Ich glaube, dass “da unten” simple Ja-Nein-Informationen verarbeitet werden, vergleichbar vielleicht mit einem binären Code. Experimente gibt es hierfür natürlich keine, allerdings ist die “Quantenverschränkung” eventuell eines der mehrfach bestätigten Naturphänomene, die dieser Informationsverarbeitung nahe kommt.

    Der sogenannte “Spiegel-” bzw. “Nivea-” oder “Kleckstest” zeigt, dass diese Tiere in der Lage sind, sich selbst im Spiegel wahrzunehmen.
    http://de.wikipedia.org/wiki/Spiegeltest
    Kühe sind toll, aber in dieser Hinsicht ist die Elster wohl tatsächlich cleverer 🙂

  26. Etwas unwohl…

    Ja, etwas unwohl wird’s mir auf jeden Fall beim Panpsychismus. Die Fragen sind ja möglicherweise interessant, spannend: Was heißt erklären, beschreiben, berechnen, erleben…? Und es lassen sich ja möglicherweise mal Zusammenhänge in einer Weise darstellen, die wir noch nicht so recht kennen. Ich lese deshalb gerne mit. Und wenn Harald Lesch als empfehlende Autorität genannt wird, dann muss was dran sein. Aber wenn jemand denkt, mit diesem Geist-Begriff sei er näher dran, religiöse und sonstige geistige Zusammenhänge zu erklären, dann wird’s mir als Theologe unwohl.
    Wer solche geistigen Prinzipien oder Strukturen (oder …?) mit Gott und Seele ff verknüpfen will, sucht mE an falscher Stelle – in physikalischen oder philosophischen Feinheiten (die als solche notwendig u berechtigt sind), anstatt zu fragen, wie Menschen mit sich selbst und anderen zurechtkommen. Und das in der ganzen Bandbreite zwischen Individual-Psychologie und politischen Fragen. Da wäre einiges zu klären; und man sollte dem nicht durch den Aufbau einer möglicherweise doch esoterischen Sonderwelt ausweichen.
    Jetzt darf ich aber meinem eigenen Blog nicht mehr ausweichen. Kaum ist man mal etwas über einen Tag unterwegs und erreicht nur sporadisch einen Computer, kabbeln sich da ein paar. Mal sehen.

  27. @P. Spät

    Ihre Behauptung, dass aus toter Materie nichts lebendes entstehen kann, ist absolut unbegründet (zumindest haben Sie bis jetzt kein Gegenargument vorhergebracht). Dafür sprechen aber einige Erkenntnisse aus der Molekülphysik (siehe z.B. Selbstorganisation und Motorbewegungen auf Molekülebene) und insbesondere in den letzten Jahren der Neurowissenschaften. Die künstlichen neuronalen Netzwerke werden immer größer und zeigen immer mehr “intuitives” Verhalten, welches durch das Lernen des Netzwerks geprägt wird.

    Zu ihren Antworten auf meine Fragen:

    1) Es ist also nur Ihr Glaube, dass die Atome geistige Eigenschaften haben?! Damit ist ja jede weitere Schlussfolgerung auf dieser These substanzlos…
    Sie sollten Ihren Glauben nicht als Tatsachen hinstellen, wie Sie es im Interview gemacht haben.

    2) Das einige Tiere sich selbst wahrnehmen ist mir bekannt. Das war auch nicht die Frage. Sie haben von der Existenz einer geistigen Eigenschaft von Atomen über die Komplexität eines Lebenwesens auf ein Bewusstsein geschlossen.
    Erstens ist es nicht klar welche Eigenschaften genau die Atomen haben sollten, zweitens ist es unverständlich was sie unter Komplexität verstehen und wie durch diese Komplexität ein Bewusstsein entstehen soll. So wie es da steht ist es ein non sequitur.

    Es gibt also zwei entscheidende Kritikpunkte:
    a) Auch wenn Ihre Behauptung von geistigen Atomeigenschaften stimmen sollte, so fehlt immer noch der Schritt zum Bewusstsein.
    b) Was Sie da beschreiben ist nichts anderes als der Reduktionismus auf geistiger Ebene und damit genau das was Sie bei den Naturwissenschaftlern kritisieren. Dadurch ist überhaupt nichts gewonnen.

  28. @P. Spät: “@ Martin B. und Martin Holzherr:
    Gute Frage! “Beweisen” kann man wohl nichts, was da unten im Kellergeschoss der Wirklichkeit vor sich geht. Da schiebt uns schon die Planck-Skala einen Riegel vor. Der Panpsychismus ist eine Theorie – und ich denke, dass sich diese Theorie aus einigen Argumenten ergibt, deren Gegenannahmen zu kusiose(er)n Theorien führt.”

    Nein, dann ist Panpsychismus keine Theorie mehr. Wenn er darauf verweist, dass wir bei der Planck-Skala nich mehr weiterkommen und wir halt Pech gehabt haben, dann immunisiert sich die Theorie selbst. Sie ist dann eine Metaphysische Aussage, aber keine wissenschaftliche mehr.

    Und um @Martin Holzherr aufzugreiffen. Wie interagiert Geist und Materie? Wenn sie miteinander interagieren, muss es eine Verbindung geben zwischen Geist und Materie. Wenn es aber diese Verbindung gibt, dann ist diese Verbindung das wesentlichere. Und dann sind wir wieder beim Materialismus.

    Da halte ich es doch lieber mit der 3-Welten-Theorie von Popper. Es existiert eine physische Welt 1, Bewusstsein/Wahrnehmung ist Welt 2, die allerdings aus der Welt 1 entsteht und das Bewusstsein, als Welt 2 generiert eine Welt 3, zB. Theorien. Alle Welten sind real, weil kausale Wechselwirkungen zwischen den Welten stattfinden.

    Trotzdem brauche ich mit dieser Sichtweise nichts Geisterhaftes erfinden, das wir aber niemals erkennen könnten. Das ist doch nur eine Ausrede mit der Planck-Länge, in Wirklichkeit wollen sie sich unangreifbar machen und jegliche Kritik abwehren. Und genau dafür wird allzu gerne die Planck-Länge wie die Quantenphysik missbraucht. Wenns schon so nciht beweisbar ist, dann verlege ich es in einem Bereich wo es niemand überprüfen kann. Billige Taschenspielertricks sind das.

  29. Rezension

    Wie ich damals nach der Rezension von Michael hier geschrieben hatte, habe ich mir das Buch gekauft und auch gelesen.
    Kurz zusammengefasst: Ich fand es sehr logisch aufgebaut, klar formuliert und größtenteils überzeugend. Auf jeden Fall gelingt der Nachweis über die logische Unschlüssigkeit der verschiedenen Spielarten des Materialismus. Dabei hätte ich mir nur ein paar Beispiele gewünscht, die konkreter ausgeführt werden, da die geistige Dimension des Geschmacks von Wein, des pochenden Schmerzes oder der Farben dem überzeugten Materialisten sonst wohl nicht klar wird. Mein Lieblingsbeispiel sind die Farben, da man damit sehr anschaulich überlegen kann: Kann ich hundertprozentig sicher sein, dass die Farbe, die mein Gegenüber als “Blau” bezeichnet, auch die Farbe ist, die ich selbst als “Blau” wahrnehme? Ich kann natürlich überprüfen, ob die Wahrnehmung einer bestimmten Wellenlänge, die diese Farbe physikalisch ausmacht, im Gehirn auf die gleiche Art und Weise verarbeitet wird, aber der springende Punkt ist, dass ich zusätzlich _annehmen_ muss, dass aus einer gleichartigen Verarbeitung im Gehirn eine gleiche subjektive Farbwahrnehmung folgt. Ich halte diese Annahme zwar durchaus für vernünftig, aber es bleibt eine Annahme.
    Nun ja, wenn man dem Buch so weit gefolgt ist, dass man die Spielarten des Materialismus verwirft (und einen Idealismus auch ohne große Diskussion nicht in Betracht zieht), dann erscheint der graduelle Panpsychismus tatsächlich als logische Alternative, die allerdings notwendigerweise spekulativ bleibt, wie im Buch auch gesagt wird. Notwendig deshalb, weil ich eine geistige Dimension, die nicht in der materiellen Dimension enthalten ist, nicht vermessen kann – unbefriedigend, aber möglicherweise Realität.
    Für mich ist das Buch ein gelungener Gedankenanstoss, an dessen spekulativen Charakter ich mich nicht störe, da ich eben von der Notwendigkeit der Spekulation überzeugt wurde. Interessant (möglicherweise auh für den Autor) finde ich auch den Vergleich zu Hoimar von Ditfurth, der ebenso wie Dr. Spät einige Male Bezug auf Pierre Teilhard de Chardin nimmt, aber in “Der Geist fiel nicht vom Himmel” zu etwas anderen Schlüssen kommt. Eine dualistische Auffassung vertritt aber auch er.

  30. @Anton Maier: Daniel Dennett

    Ja, Daniel Dennett ist ein sehr interessanter Kopf, der u.a. auch schon Arbeiten von mir zustimmend zitiert hat. Und genau dadurch zeichnet sich Wissenschaftlichkeit aus: Durch Offenheit für neue Erkenntnisse und Argumente. Wir beschimpfen einander idealerweise nicht, sondern lernen von- und aneinander.

    Sie können hier sehen, wie eine Menge Leute sachlich kontrovers diskutieren, ohne plump und persönlich beleidigend zu werden. Ich bin mir sicher, dass Sie das auch können. Oder Sie lesen halt still mit. Aber weitere Rants gehen hier gar nicht.

  31. Antworten

    Liebe Kommentatoren,

    @ alle Fragenden:
    Es taucht hier öfters der Kritikpunkt auf, dass der Panpsychismus dieses und jenes nicht beweisen könne. Streng wissenschaftlich beweisen kann keine philosophische Theorie etwas – genau deshalb ist Philosophie METAphysisch und existiert als (leider oft allzu abkapppselte) Disziplin neben den Naturwissenschaften. Philosophen können lediglich argumentieren und versuchen, die Wirklichkeit in ihrer GANZEN Fülle zu deuten. Und Theorien, die sich selbst als wasserdicht oder bewiesen darstellen, halte ich für verdächtig wenn nicht gar dogmatisch.

    @ Hermann Aichele:
    Danke für Ihr Interesse. Bei der Dringlichkeit politischer Fragen stimme ich Ihnen voll zu!
    Religiöse Aspekte klammer ich im Buch (und bislang auch hier) bewusst aus. Ich denke, dass man sowohl als Hardcore-Atheist als auch als religiös Gläubiger eine panpsychistische Position vertreten kann.

    @ Maxim:
    Mit der Selbstorganisation bin ich vertraut. Allerdings glaube ich, dass der entscheidende Impuls für ebendiese Selbstorganisation auch (nicht nur) aus den geistigen Eigenschaften kommt.

    Zu diesem Thema fällt mir ein Zitat von Hans Jonas ein, der die Richtigkeit, aber auch Begrenztheit kybernetischer Theorien recht gut auf den Punkt bringt:
    “Es ist diese Reflexion [das Nachdenken über sich selbst, PS], die der Kybernetiker seinem Gegenstande versagt. Er selbst fällt nicht unter die Bedingungen seiner Lehre. Er denkt nach über Verhalten, ausgenommen sein eigenes; über Zweckhaftigkeit, ausgenommen die eigene; über das Denken, ausgenommen das eigene. Er betrachtet von außen und versagt seinen Objekten die Vorrechte seiner eigenen reflektierenden Position. Wenn man ihn fragt, warum er es mit der Kybernetik hält, würde er, für diesen Fall einmal, nicht in kybernetischen Ausdrücken wie
    Rückspeisungen, Kreisschlingen und automatischer Kontrolle, sondern etwa so antworten: […] ‚weil ich sie für
    die Erreichung dieser oder jener Ziele für nützlich halte, und mir liegt an diesen Zielen” (Das Prinzip Leben, 1966).
    Das heißt, dass sich der Forschende nicht von seinen eigenen Prämissen ausnehmen kann – versucht er es dennoch, so gelangt er zu einem selbstwidersprüchlichen “Theoretisieren der gespaltenen Persönlichkeit” (ebd.)

    @ Sebastian Voß:
    Vielen Dank! Auch für den Hinweis auf Ditfurth.

    @ Alle, die sich an dem fundamentalen Eigenschaften “da unten” stören – und am Begriff der Komplexität:

    Ich begehe die Sünde und zitiere mich selbst aus dem Buch, da ich es ohnehin wieder so schreiben würde (S. 167ff):

    “Vergleichen wir die Buchstabenreihen „Luschthasipiel“ und „Lichtspielhaus“. Beide bestehen aus denselben Buchstaben, beide sind eine Vielheit aus Buchstaben.
    Das „Luschthasipiel“ kommt über den Status einer Vielheit
    nicht hinaus, denn die Buchstaben sind nicht sonderlich intelligent organisiert. Doch das „Lichtspielhaus“ ist mehr als die Summe seiner Teile. Denn bei diesem Wort ist die Vielheit der Buchstaben so organisiert, dass eine Einheit entsteht – nämlich ein deutsches Wort,
    dass jedermann verstehen kann und mit dem ältere Generationen das Wort „Kino“ verbinden.
    Folglich sind ein Ding oder eine Eigenschaft nicht allein dadurch komplex, dass sie eine Vielheit darstellen. Unser Körper ist eine Vielheit, die aus unzähligen Molekülen und Zellen besteht. Doch er ist zugleich eine Einheit, da die Moleküle und Zellen eng miteinander
    verknüpft und organisiert sind. Wir können nicht einfach die Zellen des Bluts und des Gehirns vom Körper wegnehmen, ohne dadurch dessen Einheit als Körper zu zerstören. Mit diesem etwas theoretischen Handwerkszeug im Gepäck können wir nun das „Komplexitäts-
    Bewusstseins-Gesetz“ formulieren:
    Je komplexer ein Ding in materieller Hinsicht ist, desto komplexer ist es in geistiger Hinsicht. Wenn also die Materie so komplex wie beim Wort „Lichtspielhaus“ organisiert ist, dann können auch parallel
    dazu die geistigen Eigenschaften komplex werden. Wenn aber die Materie so ungeordnet wie bei der Buchstabenreihe „Luschthasipiel“ ist, dann sind auch parallel dazu die geistigen Eigenschaften ziemlich
    ungeordnet und simpel. Unser Geist steht also in unmittelbarer Abhängigkeit zur Materie. Die Außen- und die Innenseite des Geiststaubs sind einmal mehr die zwei Seiten einer Medaille.”

    Seite 168ff:

    “Die IITC („Information Integration Theory of Consciousness“) des Neurowissenschaftlers Giulio Tononi ist eine der wenigen neurowissenschaftlichen
    Theorien, die naturwissenschaftliche und phänomenologische
    (das, was wir tagtäglich erleben) Beobachtungen
    miteinander zu koppeln versucht. Tononis Theorie besagt, dass ein materielles Ding in dem Maße ein bewusstes Erleben aufweist, in dem es der Informationsverarbeitung fähig ist. […] Tononi arbeitet eng mit dem Mediziner und Nobelpreisträger Gerald Edelman zusammen. Die beiden wenden sich gegen alle neurowissenschaftlichen
    Theorien, die das Auftauchen des Bewusstseins
    auf einzelne Neuronen beschränken: Nicht eine Handvoll Neuronen sind für unser bewusstes Erleben verantwortlich, vielmehr bedarf es eines sogenannten „Kerngefüges“ („dynamic core“). Neuronen, die einem solchen „Kerngefüge“ angehören, gehen für kurze Zeiträume
    intensive Verbindungen ein. In den zeitlich beschränkten Kerngefügen werden Informationen verschiedener Neuronenverbände integriert und gebündelt. Erst dann, wenn bei dieser Bündelung ein bestimmter Schwellenwert der Komplexität erreicht ist, kann unser Bewusstsein hervorgehen. Wie bei einem Laser werden verschiedene lose
    Informationen konzentriert und so ihre Komplexität derart verstärkt, dass ein „Brennpunkt“ entsteht – beim Laser der energiereiche Lichtpunkt, bei Lebewesen derjenige Ausschnitt des Geistigen, den wir bewusst erleben. […]

    Die für einen Neurowissenschaftler – zumal für einen weltweit anerkannten wie Giulio Tononi – recht ungewöhnliche Schlussfolgerung dieser Theorie hat
    eine deutlich panpsychistische Prägung:
    [im folgenden ein Zitat von Tononi] „Unter diesem Gesichtspunkt ist das Erleben, also die Bündelung von Informationen, eine fundamentale Eigenschaft, so wie auch Masse, Ladung und Energie welche sind. Daraus folgt, dass jedes physikalische System in dem Maße ein subjektives Erleben hat, wie es in der Lage ist, Informationen zu bündeln – ganz egal, aus was dieses System besteht. […] Meine Theorie behauptet, dass das Bewusstsein keine Allesoder-nichts-Eigenschaft, sondern abgestuft ist. […] Streng genommen impliziert meine Theorie, dass sogar eine binäre Photodiode nicht völlig unbewusst ist, sondern genau 1 Bit Bewusstsein erlebt. […] Im Gegensatz zum traditionellen Panpsychismus spricht meine Theorie jedoch nicht wahllos allen Dingen ein Bewusstsein zu.“

    Im Gegensatz zum traditionellen Panpsychismus geht der Graduelle Panpsychismus von einer gestuften Komplexität des Bewusstseins aus. Damit stimmt er nicht nur mit Tononis Theorie überein, er findet in dieser Theorie sogar eine neurowissenschaftliche Bestätigung
    des Komplexitäts-Bewusstseins-Gesetzes: Die Komplexität des bewussten Erlebens ist stets an die neurophysiologische (und damit materielle) Beschaffenheit gebunden. Je komplexer diese Beschaffenheit ausfällt, desto komplexer kann auch das Bewusstsein sein.
    Daher hat ein Atom vielleicht nur eine simple binäre Wahrnehmung der Wirklichkeit, indem es Ja-Nein-Informationen verarbeitet, die 1 Bit entsprechen. Die graduelle Steigerung des Geistigen korrespondiert
    mit der jeweiligen materiellen Beschaffenheit eines Dings: So findet sich im Tierreich eine gestufte Anordnung geistiger Fähigkeiten, die beim Menschen schließlich ihren (vorläufigen) Höhepunkt
    findet.”

    Pardon für das lange Eigenzitat – aber vielleicht werden die Punkte dadurch etwas klarer …

  32. @Maxim: Selbstorganisation & KIs

    Lieber @Maxim,

    ein Danke auch von mir für Ihre prägnanten Kommentare! Darf ich deshalb an zwei Stellen konkret nachfragen?

    So schreiben Sie (m.E. zu Recht) von der Selbstorganisation von Materie. Aber gerade damit wird doch ein “Selbst” impliziert, das als Potential von Anfang an vorlag und über die konkrete, äußere Form hinausweist.

    Ebenso stimme ich Ihrer Einschätzung der wachsenden “Intuition” von simulierten Neuronennetzwerken zu. Gerade das scheint mir aber dafür zu sprechen, dass wir also keinen gesonderten Schöpfungsakt brauchen, sondern zunehmend beobachten können, wie Lebendiges bzw. Geistiges aus nur scheinbar Totem erwächst. Wenn wir anerkennen, dass Leben und Geist als Potentiale schon in der Materie verwurzelt sind brauchen wir m.E. keinen Substanzdualismus mehr.

  33. Interaktion

    Wenn sich aber doch (menschliches) Bewusstsein aus der Komplexität heraus entwickelt, in welcher Weise sollen denn die einzelnen Ionen im Gehirn miteinander interagieren, um die vollständige Einheit herzustellen.
    Letzten Endes ist es ja beispielsweise so, dass beim Sehvorgang Licht auf die Netzhaut trifft und dadurch eine biochemische Kaskade auftritt, welche dann dazu führt, dass nach und nach in den entsprechenden Zellen Ionen freigesetzt werden.
    Damit kann man gut erklären, wie ein Mensch beispielsweise nach einer Tasse greift, die er gerade sieht. Die Signalkaskade endet dann eben bei den Motorneuronen am Arm etc.
    Wenn wir uns jetzt das Gehirn als Ganzes betrachten, dann entsteht eine bewusstse Wahrnehmung eines Objekts ja genau dann, wenn gewisse Signale Ionen freisetzen, welche dann wiederum Ionen freisetzen usw…
    Wie sind jetzt eben diese Ionen miteinander verbunden, dass diese ein komplexes Ganzes bilden?

  34. Interaktion

    @ PeterK:

    Die Neuronen (und damit die Ionen) gehören im Augenblick des Greifens dem beteiligten Kerngefüge (dynamic core) an, der am bewussten Erleben und den ebenfalls stattfindenden unbewussten Prozessen beteiligt ist.
    Hier ist das m.E. ganz gut erklärt:
    http://www.scholarpedia.org/…ls_of_consciousness
    Solche Kerngefüge können auch nur für ein paar Sekunden bestehen – und schwächere oder intensivere Verbindungen eingehen, je nachdem, ob wir z.B. gerade schlafen oder hochkonzentriert sind.
    Ich hoffe, Ihre Frage zielte in diese Richtung?

  35. @PeterK: Organismus besteht aus Organen,

    die einen bestimmten Zweck erfüllen. Die Frage “Wie sind jetzt eben diese Ionen miteinander verbunden, dass diese ein komplexes Ganzes bilden?” stellt sich für jedes Lebewesen, aber auch für jeden vom Menschen kreirten Automaten. Wobei wir im zweiten Fall den Schöpfer fragen können, wie das alles funktioniert, denn er hat es sich ausgedacht.

    Dass etwas Unbelebtes Belebt und die Unterscheidung zwischen konstruiert und lebendig verschwimmen kann, ist übrigens das Thema mehrerer bekannter Filme:
    Im Film AI (Artificial Intelligence) von Stanley Kubrick (verfilmt von Steven Spielberg) empfindet ein als Kind-Ersatz gebauter Roboter genau das gleiche wie ein menschliches Kind. Doch die “Adoptiveltern” wissen um die “wirkliche Natur” dieses Robots und setzen es nach Konflikten weit entfernt von “zuhause” aus.
    In Blade Runner jagt Rick Deckard, der Held des Films Replikanten, welches äusserlich nicht vom Menschen unterscheidbare Wesen sind, und erschiesst sie kaltherzig. Er erkennt die Replikanten in speziell dafür konzipierten Tests und weiss beispielsweise, dass diese Replikanten gleichartige Kindheitserinnerungen und Gewohnheiten haben, die ihnen vom Konstrukteur mitgegeben wurden. Damit werden sie für ihn zu Robots, die keinen Anspruch auf menschliche Behandlung erheben können. Am Schluss des Films kann der Zuschauer aber aus dem Zeitvertreib des menschlichen Helden Rick Deckard – dem Falten von Origami-Kranichen – schliessen, dass auch er ein Replikant ist, denn das ist genau so eine Gewohnheit, die allen Replikanten gemein ist.

  36. @P.Spät
    Danke für die schnelle Antwort.
    Ich kenne die Theorien von Edelman/Tononi und auch Ihre Dissertation.

    Was ich meinte ist vielmehr das:
    Wenn wir etwas wahrnehmen, dann finden ja, um es allgemein zu sagen, gewisse molekulare Transportvorgänge innerhalb der Neuronen und an den synaptischen Spalten ab.
    Diese Vorgänge passieren ja nicht gleichzeitig, sondern nacheinander. Wenn Neuron A erregt wird, erregt dieses Neuron B und C, durch weitere Neuronen wird wieder A erregt usw. Selbst wenn diese gleichzeitig passieren, diese Ionen selbst haben ja an sich gar keine Verbindung zueinander. Es sind ja einfach nur Bewegungen.

    Christof Koch, der ja jetzt selbst Panpsychist ist, geht ja sogar davon aus, dass eine Beschädigung des neuronalen Netzwerks für die Repräsentation der Farbe grün, sofort auch die Rotempfindung verändert, weil sich ja jetzt die Anzahl der Möglichkeiten der visuellen Differenzierung verändert, auch dann wenn das Netzwerk für grün, bei einer Rotempfindung gar nicht aktiv wäre (Die Quelle habe ich leider nicht im Kopf. Er erwähnt aber jetzt nicht, wie diese Verbindung der einzelnen Elemente zu einem Ganzen zustande kommt.

    “Solche Kerngefüge können auch nur für ein paar Sekunden bestehen – und schwächere oder intensivere Verbindungen eingehen, je nachdem, ob wir z.B. gerade schlafen oder hochkonzentriert sind.”

    Und genau das ist meine Kernfrage, was verbindet die einzelnen Elemente?

    Ich bin selbst auch kein Physikalist. Ich halte momentan die Theorie von Myers für einen guten Ansatz.
    Meines Erachtens könnte man sich das so vorstellen:
    Wenn wir uns in einem Traum befinden, der nur lange genug dauert und in diesem Traum herrschen feste Gesetze, eben genau das was wir Naturgesetze nennen. Dann könnten wir auch feststellen, dass wir nur etwas sehen, wenn wir die Augen geöffnet haben (es käme ja sonst kein Licht hindurch). Wir könnten auch die molekularen Vorgänge im Gehirn sehen und beobachten, wie sich etwas verändert, wenn wir unseren Arm bewegen, wir sprechen oder auch etwas wahrnehmen. Das Bewusstsein, Denken und auch die Erinnerungen, die sind aber alle nicht in dieser “Welt” vorhanden.
    Das Gehirn, was wir jetzt vor uns haben, wäre dann mehr eine Art Filter und den Naturgesetzen unterworfen. Es wäre nur ein Organ um mit der Umwelt wechselzuwirken, Informationen aufnehmen und auch unseren Körper anzusteuern.
    Wie man dies mit der Neurowissenschaft und insbesondere auch der Neuropsychologie mit Ihren Läsionsstudien vereinbaren kann, findet man im Buch “Irreducible Mind” sehr gut erklärt.

  37. @PeterK : irreducible mind

    Sie schreiben “Das Bewusstsein, Denken und auch die Erinnerungen, die sind aber alle nicht in dieser “Welt” vorhanden.”. Damit bekommen sie Schwierigkeiten sobald es vom Menschen konstruierte Maschinen gibt, die Bewusstsein, Denken und Erinnerungen zeigen, denn nach ihrer Vorstellung sind diese Phänomene ausserhalb der Naturgesetze. Robots, die denken, sich erinnern und ein Bewusstsein haben werden aber zweifellos den bekannten Naturgesetzen folgen. Der einzige Ausweg, der ihnen dann noch bleibt, ist die Behauptung, das Bewusstsein und Denken von Robots sei nur simuliert.

    Dies macht beispielsweise John Searle in seinem Chinese room-Argument gegen starke künstliche Intelligenz. Dabei argumentiert er, dass ein künstlich intelligentes System nur mit Symbolen hantieren könne, diese Symbole aber keine Bedeutung für das System hätten also nur etwas Formales seien. Es fehle das Verständnis. Allerdings stellt sich hier die Frage was denn Verständnis überhaupt ist.
    In der Praxis stellen wir durch Rückfragen fest, ob ein Gegenüber etwas verstanden hat. Egal wer es ist, der antwortet, sind die meisten von uns wohl überzeugt, dass in einer Situation, wo wir alle nur denkbaren Fragen stellen können sich Verständnis durch die Art der Antworten beweisen lässt oder eben widerlegt wird.
    Wenn Searle praktisch per definitionem ausschliesst, dass ein “nur Symbole manipulierendes System” etwas verstehen kann, so macht er letztlich etwas ähnliches wie viele unserer Vorfahren, die Schwarzen oder Juden aus prinzipiellen Gründen Menschlichkeit abgesprochen haben.

  38. @Martin Holzherr
    Ich meine damit natürlich nur alles auf der Ebene des Phänomenalen. Natürlich können einfache Gedächtnisvorgänge wie Habituation etc. im Gehirn stattfinden. Beim semantischen, sowie beim episodischen Gedächtnis gibt es jedoch Probleme. Was auch in o.g. Buch gut erklärt wird.
    Denken war jetzt vielleicht in dem Sinne der falsche Begriff. Man könnte es eher als mentalen (intentionalen) Gehalt des Denkens bezeichnen. Das ist ja eben genau auch in Searles Sinn. Nämlich das Symbolmanipulation in einem KI System möglich ist, es jedoch keinen mentalen Inhalt aufweist.

    Einfache Speichervorgänge, Schlussfolgern etc. kann man natürlich in einem KI System realisieren.

  39. @PeterK : Sie lehnen Funktionalismus ab

    Sie argumentieren letztlich genau wie Searle und behaupten damit: Was sich gleich verhält ist noch lange nicht gleich. Übertragen auf die Menschenwelt könnte das beispielsweise bedeuten: Zwei Experten auf einem bestimmten Gebiet zeigen genau die gleiche “Performance” (beherrschen beispielsweise eine mathematische Theorie genau gleich gut oder können gleich gut Autofahren), aber der eine ist nur ein simulierter Experte kein Echter. Und dies aus prinzipiellen Gründen, weil sie beispielsweise wissen, dass der eine Experte ein Asiate ist, der andere aber ein Europäer und nach ihrer Definition kann beispielweise nur ein Asiate ein Mathematiker sein. Ein Europäer kann Mathematikexpertise höchstens simulieren.

  40. Versuch einiger Gedankensplitter

    “ie behaupten, wir alle seien nur eine Ansammlung von Atomen, vertreten diese Auffassung aber mit emotionaler Erregung und großem, missionarischen Eifer. Und sie scheinen auch sehr motiviert zu sein, Geist und Bewußtsein Anderer zu beeinflussen.”

    Das ist so formuliert falsch. Der Physikalist erklärt zwar oft, Geist oder Bewusstsein oder auch Freier Wille seien nur Illusionen, aber er verspricht auch eine physikalische Erklärung für all diese Phänomene, in der sie auf physikalische ereignisse zurückgeführt werden.
    Ähnlich geht der durchschnittlich gebildete Mensch ja auch mit optischen Täuschungen um. Er erklärt sie, aber er hält sie nicht für das, was sie zu schein scheinen.

    “Dabei kann er aber nicht mal im Ansatz erklären, was Materie überhaupt ist: Die Naturwissenschaft zeigen, wie sich Materie verhält, aber nicht, was sie ihrem Wesen nach wirklich ist.”

    Kann mir mal jemand kurz erklären, was das Physikalische “seinem Wesen nach wirklich ist”?
    Ich denke, die Frage kann bisher niemand beantworten, vielleicht ist sie prinzipiell unbeantwortbar. Ein Positivist würde vielleicht so weit gehen, zu behaupten, die ganze Frage sei sinnlos.

  41. @Martin Holzherr
    Ich argumentiere letztlich tatsächlich so. Zwei Systeme können gleich aufgebaut sein und sich gleich verhalten, aber denoch einen Unterschied aufweisen. Eben den mentalen Gehalt.
    Natürlich fragt man sich, woher der mentale Gehalt kommt. Das weiss ich natürlich nicht. Myers nimmt an, dass wir ein größeres Bewusstsein besitzen, dies aber durch das Gehirn beschränkt wird. Seiner Meinung nach, kann man einen Blick auf das eigentliche Bewusstsein erlangen, wenn man sich Phänomene wie Savant-Syndrom oder Hypnose anschaut.
    Vielleicht entsteht Bewusstsein auch dadurch, dass ein System mit der Umwelt genügend wechselwirkt oder durch Interaktion mit anderen bewussten Wesen. Das könnte man vielleicht an Wolfskindern “testen”, wovon ich aus ethischen Gründen jedoch nichts halte.

  42. @ PeterK, Martin Holzherr, Wegdenker

    @ PeterK und Martin Holzherr:

    Das Buch von Myers kenne ich nicht – danke für den Hinweis! Ich denke, der graduelle Panpsychismus schließt – zumindest theoretisch – nicht die Möglichkeit aus, dass künstliche Gebilde (“Robots”) gewisse geistige Prozesse ausführen können. Da es nicht so sehr darauf ankommt, aus was ein Ding besteht (und am Fundament sind alle Elementarteilchen gleich und austauschbar), sondern wie es sich verhält bzw. wie es funktioniert.
    Bei der naturwissenschaftlichen Frage, was genau die einzelnen Elemente verbindet, bin ich ehrlich gesagt überfragt – vielleicht ließe sich Myers Ansatz tatsächlich integrieren. Der graduelle Panpsychismus ist (hoffe ich jedenfalls) so ausgelegt, dass er offen für Ergänzungen/Korrekturen ist.

    @ Wegdenker:

    Ich glaube auch, dass diese Frage (also was Materie/das Physikalische seinem Wesen nach ist) nicht beantwortbar ist – zumindest nicht mit dem naturwissenschaftlichen Handwerkszeug, dass mit Relationen u.ä. erfolgreich arbeitet, aber somit keine ontologischen Wesens-Fragen beantworten kann.

  43. @P. Spät, Komplexitäts-Bewusstseins-Ges.

    Macht ja nichts, dass Sie Ihr eigenes Buch zitieren, da grapsche ich mir gleich mal Ihr „Komplexitäts-Bewusstseins-Gesetz“ heraus:
    Je komplexer ein Ding in materieller Hinsicht ist, desto komplexer ist es in geistiger Hinsicht. Wenn also die Materie so komplex wie beim Wort „Lichtspielhaus“ organisiert ist, dann können auch parallel
    dazu die geistigen Eigenschaften komplex werden. Wenn aber die Materie so ungeordnet wie bei der Buchstabenreihe „Luschthasipiel“ ist, dann sind auch parallel dazu die geistigen Eigenschaften ziemlich
    ungeordnet und simpel. Unser Geist steht also in unmittelbarer Abhängigkeit zur Materie.

    Also für mich klingt das genau nach einem Hinweis *gegen* das Vorhandensein von Ihrem „Geiststaub“ oder „Samen des fundamentalen Geistigen“. (Und Leibniz’ Monaden? Meint der das gleiche wie Sie?)
    Vierzehn Buchstaben, 100 Mill. Neuronen oder ein Steinhaufen ergeben trotz viel Materie und viel „Geiststaub“ noch gar nichts. Irgendwas von Bedeutung ergeben diese Vielheiten erst, wenn sie von irgendwas Intelligentem (Mensch, Gott/Optimierungsprozess, Strichmännchen: http://xkcd.com/505/ 😉 richtig angeordnet werden. Es wird sozusagen Intelligenz hineingesteckt. (Woher diese Intelligenz kommt? Keine Ahnung, aber das spielt für diese Frage hier ja keine Rolle.)
    Klar, weil alles interessante nicht in der Materie selbst steckt, sondern in deren Beziehung. Aber wozu Sie da noch irgendwelchen „Geiststaub“ konstruieren müssen, um das zu erklären, will mir nicht einleuchten. Vielleicht als Modell oder Maß für die Beziehung zwischen Dingen, so ähnlich wie der Begriff der Information? Aber das scheinen Sie auch nicht zu bezwecken.

  44. Versuch eines Gedankenexperiments

    Nachdem ich mich mit meinen “Versuch über Gedankensplitter” bereits lächerlich gemacht habe, möchte ich noch ein kleines Gedankenexperiment beisteuern.

    Ich beginne mit folgender, provokanter These:
    Der Panpsychologismus stellt die Grundlagen der experimentellen Naturwissenschaft in Frage. Seine Konsequenzen hätten daher den Umbau weiter Teile der “normalen” Weltsicht zur Folge.

    Wenn wir postulieren, dass kleine Teilchen nicht nur “klassische” physikalische Eigenschaften haben wie Größe, Masse etc. (beziehungsweise Masse ist ja bereits als Eigenschaft der Materie abgelöst worden und wird nun auf die Interaktion zwischen Higgsteilchen und normalem Teilchen reduziert werden – mehr dazu könnte ich vielleicht beitragen, wenn ich die Formeln können würde, momentan habe ich mich nicht damit beschäftigt), sondern auch psychologische Eigenschaften wie die Gefühle, Intentionen, Reflexionsfähigkeit und ähnliches, dann ist es durchaus plausibel, dass sich Teilchen während eines Experiments anders verhalten als außerhalb eines Experiments, ja es wäre sogar möglich, dass sie sich anders verhalten, je nachdem wie der Experimentator eingestellt ist.

    Wir postulieren: Alles in der Natur erfolgt aus einem bestimmten Zweck. Die Teilchen folgen ihn. (Etwas, das man auch bei Aristoteles, Whitehead und anderen Denker findet.)
    Dann folgt, dass der Zweck eine wesentliche Eigenschaft der Natur selbst ist und man diese Eigenschaft nicht experimentell feststellen kann. Grund: Ein Experiment erfordert notwendigerweise einen künstlichen Versuchsaufbau, eine von menschen bewusst herbeigeführte Anordnung von Dingen, die so in der Natur nicht vorkommt oder vorkommen muss. Der Zweck dieser Einrichtung ist damit der Zweck, den die Menschen mit ihn verfolgen, also eine bestimmte Hypothese zu testen oder eine Theorie zu bestätigen. Dieser Zweck ist aber nicht der, den die Natur mit diesen (bzw. “ähnlichen”) Prozessen im “normalen” Ablauf verfolgt.

    Ein (fiktives!) Beispiel: Forscher entdecken, dass Ameisen, wenn sie einem bestimmten Pheromon ausgesetzt werden, wie verrückt ihre ‘Bebauungen’ verlassen und davonlaufen. Das ist der Effekt. Den hat der Forscher entdeckt als er die Ameisen diesen Pheromon ausgesetzt hat.
    Postulieren wir aber Zwecke oder Intentionen in der Natur als Eigenschaften wie Masse, Temperatur, Größe etc., dann gibt es eine wesentlich Eigenschaft dieses Pheromons, das wir noch gar nicht kennen.
    Wenn wir das Pheromon nun dazu einsetzten, Ameisen aus den Wohnräumen von Menschen zu vertreiben, dann ist der Zweck dieses Pheromons in diesen Fällen offenbar, dass man damit Ameisen von Menschen fernhält.
    Wenn wir hier halt machen, dann hieße das, der Zweck wäre nur in Bezug auf den Menschen und seinen Nutzen erklärbar. Viele, grade religiös inspirierte Denker, sind hier wirklich stehen geblieben. Ich will mich aber weiter auf diesen Pfad bewegen, den Aristoteles et al. beschritten hat.
    Nun sagen wir: “Dieser Zweck für den Menschen ist aber nicht der eigentliche Zweck. Welches ist den nun der Zweck des Pheronomeffektes nicht in Bezug auf den Menschen?”
    Die Antwort auf diese Frage kann man nicht durch Experimente erlangen, weil sie ja immer nur menschliche Zwecke haben (siehe oben, die Künstlichkeit der Anordnung). Man kann sie nur durch aufmerksames, unvoreingenommenes Naturbeobachten gewinnen.
    Dabei sehen wir z. B. (nicht vergessen, das Beispiel ist fiktiv!) das junge Ameisenköniginnen dieses Pheromon abgeben, bevor sie mit ihrer Gruppe von Wächtern, Drohnen, Arbeitern den alten Staat verlassen und eine neue Kolonie gründen. Dabei kommen noch andere Mechanismen zum Tragen, die die Arbeiter etc. veranlassen, nur ihrer “neuen Königin” zu folgen.
    Der “naturhafte” Zweck dieses Pheromoneffektes wäre also die Vermehrung der Ameisentstaaten, nicht die Ameisen aus den Wohnungen der Menschen zu verjagen.

    Ähnliche Fragen lasse sich auch für nicht-biologische Fälle stellen: “Warum scheint die Sonne?” – “Damit Photosynthese möglich ist.” – “Welches ist den nun der Zweck des Sonnenscheines nicht in Bezug auf die Photosynthese?” – …

    Ich hoffe, man kann spätestens hier entnehmen, was ich mit meinem Beitrag sagen will:
    Das Postulat von Zwecken in der Natur führt uns zur Annahme von wesentlichen Eigenschaften der Natur, die man im Experiment nicht zeigen kann, sondern durch Beobachtung erkennen muss.
    Denn der Zweck des Experimentes wäre ja immer anders, je nachdem, was der Experimentator zeigen will.
    Die Annahme von Gefühlen in der Natur führ uns zur Annahme davon, dass ein Experiment andere Ergebnisse haben kann, je nachdem, wie der Experimentator eingestellt ist. Da die Teilchen dann – durch die Gefühle des Experimentators beeinflusst (ich gehe einfach von Empathie aus) – auch anders reagieren. Das ähnelt in Übrigen der Situation, in der sich Parapsychologen und davor teilweise Alchemisten befanden, die auch postulier(t)en, dass das Psychische die Natur (grich. Physik) beeinflussen oder gar verändern kann.

    Demnach besteht ein Widerspruch zwischen der heute üblichen, naturwissenschaftlichen Vorgehensweise und der Annahme von Zwecken oder Gefühlen in der Natur.
    Ich will jetzt nicht sagen, dass damit der Streit bereits gegen die Annahme von Zwecken oder Gefühlen in der Natur entschieden worden ist, aber man sollte sich der Konsequenzen dieser Annahme bewusst sein. Könnte apriori durchaus sein, dass uns die Naturwissenschaften immer nur eine Teilwahrheit zeigen konnten, die aber zur technischen Beherrschung und teilweisen Erklärung der Natur reicht.
    Möglicherweise müsste man teile der Wissenschaft verändern, um dieser Annahme gerecht zu werden. Das wäre dann allerdings ein größeres Projekt.

    Ich hoffe, mein Gedankenexperiment eröffnet (auch durch Kritik an ihn) zumindest neue Perspektiven auf das Problem. 😉
    Schönen Tag noch.

    P.S.: Ich weiß, dass ich meinen obigen Einwand auf das Gedankenexperiment selbst noch nicht hinreichend eingesetzt habe.

  45. @ Daniel und Wegdenker

    @ Daniel:

    Als Geiststaub bezeichne ich keine ontologische “Extrawurst”, sondern Materie+Geist, also die Materie, wie wir sie kennen und naturwissenschaftlich beschreiben (ihre Außenseite) samt ihrer von uns erlebten Innenseite.
    Den Geiststaub brauche ich also, weil ich sonst nur die nackte, mathematisierbare Materie habe, von der der Physikalist spricht.

    Ja, die Monaden von Leibniz kommen dem ziemlich nahe! Mit dem großen Unterschied, dass Leibniz die Monaden “fensterlos” denkt und daher die prästabilierte Harmonie braucht, während ich (wie Alfred N. Whitehead und Pierre Teilhard de Chardin) davon ausgehe, dass sie untereinander stark agieren und somit “Fenster” haben.

    @ Wegdenker:

    Klingt spannend – und ist überhaupt nicht lächerlich! Davon lebt die Philosophie ja 🙂
    Zunächst glaube ich allerdings NICHT, dass der Panpsychismus in irgendeiner Weise die Erkenntnisse der Naturwissenschaft berrührt, verletzt oder außer Kraft setzt! Er kann höchstens unser metaphysisches Gesamt-Weltbild verändern. (Was ist eigentlich der Grund dafür, dass Sie von einem “Panpsychologismus” sprechen?)

    Andererseits KANN es durchaus sein, dass die Teilchen im Mikrokosmos mehr drauf haben als wir Ihnen gemeinhin zutrauen – ich denke da gerade an die “Quantenverschränkung”.

    Sie schreiben:
    “Alles in der Natur erfolgt aus einem bestimmten Zweck.”
    Ich glaube (und schreibe in dem Buch) durchaus, dass es Zwecke in der Natur gibt. Insofern Zustimmung. Aber ich habe größte Bedenken, wenn diese Zwecke jenseits/über der Natur schweben, wenn also die Natur einem gottgewollten Plan folgt o.ä., wo wir schnell bei einem Intelligent Design landen könnten.
    In welche Richtung zielen Ihre postulierten Zwecke ab?

  46. @Michael Blume

    Die Selbstorganisation ist kein Voodoo-Zauber und hat nichts geistliches. Sie ist eine direkte Folge von physikalischen Gesetzen und lässt im Rahmen der Physik auch vollständig erklären. Dafür ist überhaupt keine neuen geistigen Eigenschaften nötig (Welche das sind wurde immer noch nicht beantwortet…).
    Ein Wassertropfen ist doch nicht deswegen rund, weil er das möchte, sondern weil zwischen den Wassermoleküle Bindungskräfte vorhanden sind, die sich gegenseitig so lange bewegen, bis ein Gleichgewicht herrscht.

    “dass wir also keinen gesonderten Schöpfungsakt brauchen, sondern zunehmend beobachten können, wie Lebendiges bzw. Geistiges aus nur scheinbar Totem erwächst. Wenn wir anerkennen, dass Leben und Geist als Potentiale schon in der Materie verwurzelt sind brauchen wir m.E. keinen Substanzdualismus mehr.”

    Ich bin nicht ganz sicher was Sie mit dem Wort “scheinbar” sagen wollen. Ein Siliziumchip ist tot oder meinen sie da schlummern schon irgendwelche geistigen Eigenschaften?
    Ansonsten die Physikalisten genau das: man braucht keine “geistliche” Schöpfung, das Bewusstsein hat die Materie als Unterbau.
    Physikalisten brauchen keinen “Substantzdualismus”. Diese Annahme ist unbegründet, da sie nicht empirisch ist und bringt keine weiteren Erkenntnisse, sondern schafft jede Menge neue Probleme, wie z.B. Wie interagiert der Geist (was immer das auch sein soll) mit der Materie? Das letztere sollte man direkt beobachten können, hat man aber nicht.

  47. Noch mehr Fragen

    e)
    “Natürlich haben naturwissenschaftliche Erkenntnisse ihre Gültigkeit, doch ihr Gültigkeitsbereich ist eben beschränkt”

    Was ist der Gültigkeitsbereich des Panpsychismus? Welche Aussagen über die Welt trifft er überhaupt – über die hinaus, dass Geistiges überall dort anzutreffen ist, wo auch Materielles ist?

    j)
    “Erst dann, wenn ein Ding in materieller Hinsicht ausreichend komplex ist, nimmt auch seine geistige Seite komplexere Formen an.”

    “Das Geistige ist […] ein natürliches Phänomen wie Ladung, Masse und Spin auch.”

    Wird der Panpsychismus durch solche Aussagen nicht zwangsläufig zu einer materialistischen Theorie? Oder gibt es Geistiges, im Gegensatz zu Ladung und Masse, auch ganz ohne Materie?

    k)
    [Das Geistige] “lässt sich nicht durch Formeln einfangen.”

    Gravitation und Elektromagnetismus lassen sich an der Materie empirisch als Eigenschaften messen und in Formeln relational beschreiben (“einfangen”), Geistiges nicht. Ist der Panpsychismus nicht einfach eine materialistische Theorie mit einem Eigenschaftsdualismus (einfangbar / nicht einfangbar), aber eigentlich identischer Ontologie?

    o)
    “Der Emergentist glaubt, dass aus dem rein materiellen Hirn plötzlich geistige Eigenschaften “auftauchen”. […], dann kommen die Lebewesen und mir nichts dir nichts macht es “plopp” und der Geist ist da.”

    Wie soll ich dieses “plötzlich” verstehen, phylogenetisch und/oder ontogenetisch? Bei welchen Lebewesen hat es denn “plopp” gemacht? In welchem Entwicklungsstadium des Menschen macht es “plopp”? Welcher Emergentist glaubt das oder beschreibt das so, können Sie mir da einen Autor nennen?

    r)
    “Wir sind keine Waschmaschinen, deren Funktionen wir lückenlos in einer Anleitung beschreiben könnten.”

    Warum versucht dann der Panpsychismus solche Lücken zu schließen, die die Naturwissenschaft hinterlässt, und versucht gerade doch so eine Anleitung zu schreiben?

    Gibt es bei Ihnen persönlich ein metaphysisches Bedürfnis hinsichtlich des Lückenfüllens? Kann der Panpsychismus Trost spenden, ähnlich wie andere metaphysische Theorien – die Religionen – dies angeblich machen?

    -)
    Was ginge der Welt mit dem Ende des Panpsychismus verloren?

  48. @Maxim

    Schade, dass Sie offenbar lieber mit starken Begriffen um sich werfen, als sich auf das Abenteuer Denken einzulassen. Was soll es denn genau heißen, Selbstorganisation sei kein Voodoo-Zauber? Ein sachliches Argument ist es nicht. Definieren Sie uns doch bitte einfach “Selbstorganisation” – woher kommt Ihrer Überzeugung nach das Selbst und wer organisiert da wen?

    Und ja, auch ich gehe davon aus, dass alle geistigen Phänomene einen physischen “Unterbau” haben (das war mein Promotionsthema). Die hier diskutierte Frage ist doch – woher stammen dann die oberen Stockwerke (Leben, Geist)? Waren sie schon immer in der Materie veranlagt oder wurden sie (woher & wie?) zugegeben?

    Zuletzt: Ja, Panpsychisten gehen davon aus, dass auch in Siliziumchips Potentiale von Leben und Geist schlummern, also künstliche Intelligenzen keinen immateriellen Schöpfungsakt (oder “Voodoo-Zauber”) brauchten. Sie selbst hatten doch auf die zunehmend beeindruckenden Fähigkeiten von Computernetzen verwiesen – die auch mich nachdenklich machen.

    Überzeugen will ich Sie zu gar nichts, aber zum Mitdenken einladen. Es liegt an uns, ob wir Begriffe wie Selbstorganisation, Neuronennetze etc. durchdringen oder uns mit heiteren Voodoo-Vergleichen begnügen. Auch Atomhaufen haben eine Wahl, oder!? 😉

  49. @Michael Blume
    Ich frage nur etwas genauer nach, weil es alles so schwammig formuliert ist und genaue Festlegungen vermieden werden. Bis jetzt hat der Herr Spät meine Fragen nicht explizit beantwortet, deswegen gehe ich davon aus, dass er keine Antworten kennt. Wie er bereits angedeutet hat, ist alles was er hier vorgetragen hat nur sein persönlicher Glaube (“Ich glaube ..”). Was nun mal wirklich kein Argument ist. Der Schritt zum Voodoo-Zauber ist da gar nicht so weit.

    Zur Selbstorganisation: Ein einfaches Beispiel aus der Makrowelt: Nehmen Sie zwei magnetische Kügelchen und legen sie nebeneinander hin. Was beobachten Sie? Was passiert wenn man 5 oder 10 davon nimmt. Wie ordnen sie sich an? Wer oder was organisiert sie an?

    Zu den KNN: Die Fähigkeiten von KNN erklären sich aber ohne Annahme von geistiger Eigenschaften in Silizium. Damit ist es kein Argument für Panpsychismus.

    Ich lasse mich gerne auch von fliegenden Teetassen überzeugen, wenn harte Argumente hervorgebracht werden. Bis jetzt habe ich aber noch keine gesehen.

  50. @Maxim

    Okay, in der Suche nach Begriffsklarheiten stimmen wir überein – genau deswegen lade ich ja gerne hin und wieder hier zu auch philosophischen Debatten. 🙂 Die Auseinandersetzung mit verschiedenen, philosophischen Positionen birgt m.E. vor allem die Chance, vermeintlich Selbstverständliches (da ist das Selbst schon wieder!) neu zu betrachten. Wie eben den Begriff der “Selbstorganisation”! Das Video schaue ich mir gerne an und melde mich dann wieder!

    Besten Dank & Grüße!

  51. @Spät @Blume

    Spät:
    Entweder geht der Emergentist davon aus, dass alles materiell ist: Dann hat er den Salat, den ich im Interview versucht habe zu beschreiben. Oder er geht davon aus, dass das Geistige “irgendwie” in der Wirklichkeit vorhanden ist. Dann kann er Panpsychist, Idealist, Dualist oder anders sein. Ich sehe keinen (logischen) Ausweg aus diesem Dilemma.

    Blume:
    Und zu Emergenz: Ich kann nicht erkennen, wie sich Emergentismus und Panpsychismus notwendig ausschließen sollten. Die Emergentistin erkennt doch m.E. zunächst einfach nur an, dass neuartige (System-)Eigenschaften im Zeitverlauf beobachtbar auftreten. M.E. ist damit doch noch gar keine Letztaussage verbunden, ob diese schon immer (“panpsychisch”) in der grundlegenden Materie “verwurzelt” oder durch zufällig-evolutionäre Prozesse, durch göttliches Eingreifen o.ä. hinzugefügt werden. Mir scheint es ein Vorzug des Emergentismus zu sein, dass er an dieser Stelle bis zum Auftauchen neuer Erkenntnis aus guten Gründen stehenbleiben und agnostisch sein kann. Oder habe ich da etwas völlig falsch verstanden?

    Ich schrieb bereits, dass die verschiedenen Zugänge zur Realität sich nicht in Widersprüche zueinander verwickeln dürfen, denn als Minimalkonsens müssen wir doch annehmen, dass es nur *eine* Realität gibt. Wenn z.B. aus physikalischen und biologischen Erkenntnissen geschlussfolgert werden kann, dass kein Mensch aus dem Stand zehn Meter hoch springen kann, dann kann keine Theorie des Geistes hier Gegenteiliges schlussfolgern – wenn nicht eine der Theorien (Physik, Biologie, Geist) dadurch falsifiziert werden soll.

    Das bietet die Gelegenheit, die Idee des Emergentismus auf eine solidere Grundlage zu stellen. Der Inhalt des Begriffs darf nicht gegen physikalische Grundlagen verstoßen, es sollte aus dieser Erklärung aber andererseits deutlich werden, dass bestimmte Vorgänge bzw. Beobachtungen nicht auf die Physik reduziert werden können. Ich habe das hier (und mehr für mich) schon einmal ausführlich dargelegt, im Folgenden die Kurzversion:

    Die Grundthese des Emergentismus lautet: Das Ganze ist mehr als die Summe seiner Teile. Dazu ein Beispiel: Angenommen ein System setzt sich aus den Teilen A und B zusammen, wobei A wiederum aus A1 und A2 besteht, B aus B1 und B2. Innerhalb von A wirkt A1 auf A1 und A2, A2 wirkt auf A2 und A1. Selbiges gilt für B. In dem zusammengesetzten System wirken aber A1 auch auf B1 und B2, sowie die anderen Komponenten ebenfalls auf die jeweiligen Teile des anderen Teilsystems. Man kann einfach auszählen und erhält, dass es im Gesamtsystem 16 Wechselwirkungen gibt, man bei einer getrennten Analyse der Teilsysteme A und B man aber nur 2×4=8 davon untersuchen kann.

    Die Schlussfolgerung: Bei einer Reduktion der Untersuchung auf Teilsysteme gehen Wechselwirkungen verloren, die prinzipiell nur im Gesamtsystem vorhanden sind. Um alles über das Gesamtsystem zu erfahren, muss man auch das Gesamtsystem untersuchen.

    In den Wissenschaften finden eine Reduktionen auf verschiedenen Ebenen statt:
    – Anstelle des Menschen einschließlich seines sozialen Umfelds wird sein Gehirn untersucht.
    – Anstelle des Gehirns einzelne Nervenzellen.
    – Anstelle einer Zelle einzelne Organellen.
    – Anstelle der Organellen chemische Verbindungen.
    – Anstelle von Molekülen Atome und deren Zusammensetzung.

    Auf jeder Analyseebene gewinnt man zwar wichtige Erkenntnisse, zugleich gehen aber Informationen über die Systeme verloren, die aus diesen Teilen zusammengesetzt sind. Oder wie zum Beispiel in dem bereits verlinkten Beitrag: Aus der Untersuchung des Aufbaus eines Kohlenstoffatoms können wir zwar (physikalische) Einschränkungen für Systeme ableiteten, die sich aus solcherart Atomen zusammensetzen lassen, über die tatsächlich realisierte Struktur dieser Systeme “wissen” die Atome aber nichts. Also z.B. wie sie in einen Tisch oder ein Gehirn bilden können.

    Eng damit verknüpft ist der Begriff der Abwärtsverursachung, der meiner Meinung nach einen entscheidenden Unterschied zwischen dem Emergentismus und dem Physikalismus (Reduktionismus) bildet: Die Wechselwirkungen des zusammengesetzten Systems (also die Informationsverarbeitung) kann sehr wohl die Wechselwirkungen seiner Teile beeinflussen. Das ist die Antwort auf etwas, dass beim Physikalismus entweder durch ein Dogma ersetzt oder ganz geleugnet wird: Wieso empfindet ein kohlenstoffbasiertes Wesen mit Bewusstsein etwas, während ein kohlenstoffbasierter Tisch dies nicht tut? – Es ist schlicht die andere Art der Wechselwirkung derselben Atome in beiden Systemen. Mit dem ontischen Status der Begriffe “Atom” oder “Gott” hat das nichts zu tun, eine Diskussion dieser metaphysischen Begriffe muss man dazu überhaupt nicht führen.

  52. @Blume

    Bitte in meinem letzten Kommentar die beiden ersten Absätze als Zitate von Herrn Spät und von Ihnen kennzeichnen. Das gesetzte cite-Tag wurde dort leider verschluckt.

  53. Geist verstehenAbstraktion verstehen

    Der Panpsychismus entspringt dem Unvermögen zu verstehen, wie sich Denken entwicklen kann. Dabei sind wir gar nicht allein mit unseren geistigen Fähigkeiten. Wir haben nur die höchsten geistigen Fähigkeiten in einem zu beobachtenden Kontinuum geistiger Leistungsfähigkeit. Einfache Nervensysteme stehen oft unmittelbar zwischen Sensoren und Aktuatoren (Flagellen, Muskeln, etc), während komplexere Nervensysteme zu Tieren gehören, die sich erinnern und die lernen können.
    Raubtiere benutzen bereits Verhaltensmodelle ihrer Beute und Modelle der Jagdumgebung. Wenn ein Eisbär auf Robbenjagd unter der Eisscholle auf dem das Beutetier ruht, durchtaucht um auf der anderen Seite blitzschnell aus dem Wasser zu schnellen und seine Beute so zu überraschen, so tut er dies nicht einfach aufgrund eines Reiz-Reaktionsschema oder weil ihm dieses Verhalten angeboren ist, sondern weil er die Jagdsituation versteht und sich von mehreren Handlungsoptionen die erfolgsversprechenste aussucht. Soziale Tiere haben auch ein Modell, eine Theory of Mind ihrer Artgenossen. Was man in obiger Referenz liest, nämlich “It is typically assumed that others have minds by analogy with one’s own, and based on the reciprocal nature of social interaction, as observed in joint attention,[4] the functional use of language,[5] and understanding of others’ emotions and actions” gilt nicht erst für Menschen, sondern auch für andere Primaten. Das zeigen Beobachtungen unter Schimpansen, die nicht selten Tauschhändel der Art “Lausen gegen …” und “Sex gegen Futter und …” eingehen. Somit gilt bereits für höhere Tiere: “Having a theory of mind allows one to attribute thoughts, desires, and intentions to others, to predict or explain their actions, and to posit their intentions.”
    Eine solche Theory of Mind impliziert aber auch Selbstbeobachtung, Reflexion.
    Wir bewegen uns also in einer Hierarchie von Abstraktionen geistiger Aktivität. Auf der untersten Stufe gibt es Reflexe, auf einer höheren Stufe Denken mit Modellen und schliesslich Modelle für Artgenossen und sich selbst. Es ist nicht verwunderlich, dass sich bei der Ausbildung dieser Abstraktionen auch Phänomene wie Sprache ausgebildet hat, denn schon die geistige Prozesse wie Pläne schmieden, was ebenfalls schon bei Affen vorkommt, benötigen symbolisches Denken welches sehr sprachnah ist.

    Höhere geistige Prozesse haben viel Rekursivität in sich, wo also – unter anderem – über das Denken nachgedacht wird. Das Buch Gödel, Escher, Bach von Douglas Hofstadter sieht selbstbezügliche Schleifen sogar als Grundthema der Kognition: “Gödel, Escher, Bach: An Eternal Golden Braid (commonly GEB) is a 1979 book by Douglas Hofstadter, described by his publishing company as “a metaphorical fugue on minds and machines in the spirit of Lewis Carroll”

    Der hohe, leider durch fehlende Einfachheit nicht für alle eingelöste Anpsruch des Buches lässt sich so fassen:

    ” Through illustration and analysis, the book discusses how self-reference and formal rules allow systems to acquire meaning despite being made of “meaningless” elements. It also discusses what it means to communicate, how knowledge can be represented and stored, the methods and limitations of symbolic representation, and even the fundamental notion of “meaning” itself.”

  54. @Maxim

    Das Video habe ich mir nun angeschaut und finde es durchaus schön. Nur: Irgendeine Erklärung für das Auftauchen psychischer Zustände (also das Thema unserer Debatte) konnte ich nicht erkennen. Vielmehr doch eines: Der Magnetismus war den gezeigten Artefakten bereits eigen – “deshalb” konnten sie sich erst zu Mustern verbinden. Wenn ich den Panpsychismus richtig verstanden habe, ist das doch auch deren Aussage: Grundlegende psychische Funktionen sind Materie bereits eigen – deshalb kann sie z.B. Bewußtsein entfalten.

    Ebenso möchte ich bei Ihrer Aussage zu künstlicher Intelligenz nachfassen. Sie schreiben: Die Fähigkeiten von KNN erklären sich aber ohne Annahme von geistiger Eigenschaften in Silizium.

    Nur einmal angenommen, künftige KNNs würden Bewußtsein, ein geistiges Innenleben entfalten – woher würden diese Eigenschaften denn dann Ihres Erachtens stammen, wenn nicht aus der grundlegenden Materie? Und finden Sie, dass wir das heute schon ausschließen können? Meinen Sie (und @Patrick Spät), dies könnte ein Weg zu einer empirischen Überprüfung des Panpsychismus sein?

    Mit Dank & gespannten Grüßen!

  55. @Köppnick

    Vielen Dank für Ihre Darstellung, der ich mit großem Interesse gefolgt bin! (Und die Formatkorrekturen habe ich wie erbeten auch vorgenommen.)

    Mir stellt sich aber eine Frage: Unsere Form der Kommunikation über das Internet, dortselbst wiederum vermittelt über Symbole wie Zahlen und Buchstaben, wäre noch unseren Vorfahren vor wenigen Jahrzehnten als gröbster Verstoß gegen physikalische Regeln erschienen, oder? Wir sind nicht in einem Raum, sind uns nie (zumindest mir nicht bewusst) begegnet, haben uns auch nie gehört – und tauschen dennoch Geistiges (!), nämlich Argumente und Fragen aus. Teilhard de Chardin hätte das wohl Noosphäre genannt, mich würde aber wirklich interessieren, wie Online-Kommunikation in Ihr (m.E. sehr interessantes!) Paradigma passt.

  56. @Michael Blume

    Zu dem Video: Im Gegensatz geistigen Atomeigenschaften ist Magnetismus beobachtbar 😉 Der Schritt zwischen den vorhandenen magnetischen Kräften (auf der Quantenebene) und daraus folgender Selbstorganisation im Makroskopischen ist vorhanden und kann mathematisch und somit auch logisch nachvollzogen werden. Dieser Schritt fehlt beim Panpsychismus komplett. Es wurde zwar das Wort “Komplexität” erwähnt, aber was das ist bzw. wie sie definiert ist, kommt überhaupt nicht rüber. Dieses Wort alleine ist zu allgemein und sagt eigentlich nicht viel aus. Je nach Themengebiet muss es genau festgelegt werden.

    “Nur einmal angenommen, künftige KNNs würden Bewußtsein, ein geistiges Innenleben entfalten – woher würden diese Eigenschaften denn dann Ihres Erachtens stammen, wenn nicht aus der grundlegenden Materie? Und finden Sie, dass wir das heute schon ausschließen können? Meinen Sie (und @Patrick Spät), dies könnte ein Weg zu einer empirischen Überprüfung des Panpsychismus sein?”

    Das ist natürlich nur mein Wissensstand, aber diese Eigenschaft würde aus der Vernetzung von vielen einzelnen (Neuronen)Zuständen kommen.
    Wie genau diese Vernetzung funktionieren soll/wird, ist ein Gegenstand der Forschung (siehe “Human Brain Project” oder die Entwicklung von Hardware-KKNs im Rahmen des “BrainScaleS” Projekts).

    Bereits ein einfaches KNN aus 10 Neuronen kann z.B. handgeschriebene Zahlen erkennen, obwohl ein Neuron für sich alleine nur logische Operationen beherrscht. Damit das KNN diese relativ anspruchsvolle oder soll ich besser schreiben intuitive Aufgabe lösen kann, muss es lernen. Man muss dem Netz anhand von Fallbeispielen zeigen was falsch und was richtig ist (Vergleiche mit dem menschlichen Lernverhalten sind erlaubt ;)).
    Ein Beispiel für die Mächtigkeit von KNNs: Herr Spät hat angemerkt, das man den Geschmack von Zucker nicht verstehen kann. Da wäre ich mir nicht so sicher: http://www.spiegel.de/…geruchssinn-a-708688.html
    Ich mir sicher, dass die Eigenschafen der Atome da gar nicht entscheiden sind, wofür ein Software-KNN mit einem Bewusstsein ein direkter Beweis wäre. Damit wäre die Idee des Panpsychismus überflüssig.

    Wenn ein Software-Bewusstsein entsteht, dann kann man es bis zum Umfallen studieren und möglicherweise auch mathematisch beschreiben. Damit hätte man eine lückenlose mathematische Beschreibung von den Elementarteilchen bis zu einem Bewusstsein.

    Meiner Meinung nach ist es nur eine Frage der Zeit bis es ein bewusstes KKN gibt, denn es spricht aus naturwissenschaftlicher Sicht nichts dagegen.
    Bis es aber soweit ist, können die Philosophen sich weiter die Zungen wund reden 😀

  57. Interessante Diskussion

    “Unsere Form der Kommunikation über das Internet, dortselbst wiederum vermittelt über Symbole wie Zahlen und Buchstaben, wäre noch unseren Vorfahren vor wenigen Jahrzehnten als gröbster Verstoß gegen physikalische Regeln erschienen, oder?”

    Das versteh ich jetzt nicht, Herr Blume. Wieso sollte das ein gröbster Verstoß gewesen sein? Die Grundlage dieser Art von Kommunikation ist die Schrift, die es schon seit einigen tausend Jahren gibt. Das Internet ist ein technisches Hilfsmittel wie es früher die Schreibmaschine und noch früher der Brief war. Ja, die Geschwindigkeit hat sich verändert, aber auch mit den erwähnten früheren Hilfsmitteln ist man sich nicht von Angesicht zu Angesicht gesessen und konnte damit die bei menschlicher Kommunikation eigentlich nicht unwichtigen Merkmale wie Gestik, Mimik, auch Aussehen, Präsentation u.ä. wenn, dann nur rudimentär erfassen. Die physikalische Grundlage für diese neue Art der Kommunikation liegt im wissenschaftlichen Fortschritt, die es ermöglicht hat die Buchstaben der Sprache auf 0 und 1en herunterzubrechen und sie über große Distanzen blitzschnell zu schicken, um sie am gewünschten Zielort wieder zusammenzusetzen, möglichst so wie es verschickt wurde.

    Und wie ich schon sagte, all das baut auf die Erfindung?, Entdeckung? der Schrift auf. Sie ist sozusagen die schon etwas abstrakte Grundlage dessen, was nachher kam und darauf aufbaute.

  58. @Maxim

    Ja, bei physischen Artefakt Magnet können wir Magnetismus beobachten – und beim Menschen @Maxim geistige Tätigkeit! Beides beobachtbar da, mit Auswirkungen auf die physische Umwelt! Und wir nehmen beide an, dass die Eigenschaften “magnetisch” bzw. “diskussionsfreudig” eine materielle Grundlage haben, nicht wahr?

    Wobei Sie, wenn ich Sie richtig verstehe, die “höheren” (emergenten?) Eigenschaften eher in den materiell bedingten “Netzen” als den Materiepartikeln selbst vermuten würden.

    Finde ich sehr cool – und zeigt doch, dass wir uns über die philosophischen Debatten langsam in empirisch testbares Terrain vorarbeiten. Also, mir bringt das was! 🙂

    Ihnen nochmal Danke für die hochkarätigen Kommentare!

  59. @Carlo: Schrift

    Yo, genau da wollte ich hin: Mit Symbolen (Kunst, Schrift etc.) können verstehende (also: geistig aktive!) Lebewesen über Zeit und Raum hinweg kommunizieren – ohne dass wir annehmen müssten, dies widerspreche den Naturgesetzen. Es entfalten sich auf physikalisch-chemisch-biologisch-psychisch-soziolultureller Grundlage also auch völlig neuartige Phänomene und Eigenschaften – und wir können noch nicht einmal wissen, welche noch kommen…

    Lasset uns staunen, was aus vernetzten Atomen so alles entsteht!!! 🙂

  60. “Ja, bei physischen Artefakt Magnet können wir Magnetismus beobachten – und beim Menschen @Maxim geistige Tätigkeit! Beides beobachtbar da, mit Auswirkungen auf die physische Umwelt!”

    Wie bereits gesagt: Im zweiten Fall gibt es keine geschlossene Kausalitätskette.

  61. @Maxim
    Das KNN kann natürlich Muster repräsentieren, je komplexer das Netzwerk, desto komplexer die Muster.
    Am Ende muss aber immer ein Anwender stehen, der dieses Muster interpretiert. Das KNN repräsentiert, kann jedoch den semantischen Gehalt dessen was es repräsentiert nicht erfassen. Deswegen kann das neuronale Netz auch nicht bewusst werden.
    Sollte es softwarebasiert möglich werden, dann könnten wir ja auch einfach unser Gehirn nachbilden und es gäb uns zwei mal. Wenn wir dann das neuronale Netz reizen, müssten wir ja in diesem Moment Schmerz spüren.

  62. Noch als Anmerkung:
    Ich hatte ja dieses Buch “Irreducible Mind” angesprochen, welches die Theorie von Myers behandelt.
    Einer der Autoren ist ein Psychiater, der intensiv auf dem Gebiet der Spracherkennung durch neuronale Netze gearbeitet hat. Also nicht nur ein Psychiater auch durch und durch ein kognitiver Neuropsychologe.

    Er vertritt und begründet in diesem Buch die These, dass die kognitive Psychologie mit ihren neuronalen Netzen das Bewusstsein niemals erklären kann, da diese immer einen Beobachter voraussetzt.

  63. @Michael Blume

    Ein meiner Ansicht nach illustrativeres Video zur Selbstorganisation wäre dieses hier: [Video Feedback Loops Lead to Self Organization]

    Weiters sei noch auf ein neueres Buch von Douglas Hofstadter, I Am a Strange Loop, verwiesen. Man beachte, dass sich bei diesem Hofstadter-Link wiederum ein weiterfühender Link zu “Optical feedback” findet, was dann auch eine thematische Verknüpfung zu obigem Video schafft.

    In Anspielung auf den vitalistischen élan vital spricht Hofstadter übrigens dort von élan mental hinsichtlich jener Vorstellungen, die dem Geistigen eine eigene Form von Stofflichkeit zuerkennen wollen. Diese Bezeichnung trifft’s recht gut, wie ich meine.

  64. “Wer soll das bezahlen?”

    “Ein Notfall – Wir brauchen einen Philosophen!” Wie reagieren Sie?”

    Philosophie, Kunst und Kultur, sind in dieser Welt- und “Werteordnung” der Hierarchie von und zu materialistischer “Absicherung” leider nur Schmuck, für den scheinbaren Glanz einer Kommunikation in Überproduktion von systemrationalem Kommunikationsmüll.

    Vor Kurzem erzählte mir ein Mensch aus dem Medizinsektor, daß an den Universitäten Yale und Harvard die Geistheilung langsam aber sicher in die Phase der Vermarktung gebracht wird – vielleicht zehn / zwanzig Jahre noch.

    Da dachte ich mir: Wenn das stimmt, dann paßt das mal wieder. So schizophren und widersinnig zum Geist der uns alle im selben Maß durchströmt und gleich macht, funktioniert unsere “individualbewußte” Vorstellungswelt. Nur der wettbewerbsbedingte Markt soll heilen, wo geistig-heilendes Selbst- und Massenbewußtsein den Markt und all seine stumpf-, blöd- und wahnsinnigen Symptomatiken ad absurdum führen könnte / sollte 🙂

  65. Informationsaustausch per Internet

    @Blume/Köppnick
    Ob man Informationen per Sprache, Gesten oder Text/Internet austauscht ist im Prinzip gleich. D.h. wir machen jetzt nichts anderes als das, was seit Jahrtausenden geschieht.

    Der Austausch geschieht wie folgt: Die Information von Person A ist nichts anderes als ein Muster. Wenn Person B dieses Muster wahrnimmt, dann werden beim Erkennen in B´s-Gehirn die für dieses Muster zuständigen Gehirnareale aktiviert – das Muster wird nun bewusst wahrgenommen. Und Person B versteht die Information von A.
    Im eigentlichen Sinne ist dies kein Tausch, sondern es handelt sich um zwei unabhängige Aktivitäten.

    Wenn A und B ein gemeinsames kulturelles Allgemeinwissen haben, dann kann B die Information von A verstehen. Fehlt z.B. diese gemeinsame Wissensbasis, so kann die Information von A nicht in dem Sinne weitergegeben werden, wie sie von A gemeint war – B fehlt das Verständnis.

    Ein schönes Beispiel hierfür ist das Lesen/Hören religiöser Texte unter dem Kernspintomographen: bei einem gläubigen Menschen werden andere Bereiche des Gehirns aktiviert als bei einem Atheisten.

  66. “Das KNN repräsentiert, kann jedoch den semantischen Gehalt dessen was es repräsentiert nicht erfassen. Deswegen kann das neuronale Netz auch nicht bewusst werden.”

    Der erste Satz stimmt einfach nicht. Eine Rückkopplung auf die eigenen Strukturen und Aufteilen des KNNs in mehrere Bereiche ist zum Beispiel ein Gegenbeispiel. Aber wenn der erste Satz auch stimmen würde, so folgt daraus immer noch nicht Ihre zweite Aussage. Ein Gegenbeweis: Ein Mensch kann seinen eigenen semantischen Gehalt des Gehirns nicht erfassen (oder wissen Sie welche Neuronen gerade bei Ihnen aktiv sind?) und doch hat er ein Bewusstsein.

  67. Bewußtsein begrenzt???

    “… Kernspintomographen: bei einem gläubigen Menschen werden andere Bereiche des Gehirns aktiviert als bei einem Atheisten.”

    “Ein Notfall – Wir brauchen einen Philosophen!”

    Na hier paßt der Witz doch prima, denn Neuro-Biologen & Co. wissen diese Bereiche auch nur vage zu deuten – verschiedene Lust- und Suchtzentren!?

  68. @KRichard:Sprache/Denken ist mehrkanalig

    Die menschliche Sprache transport vieles aufs Mal, enthält redundante aber auch kontextabhängige Elemente, stellt eine Vielzahl Worte ähnlicher Bedeutung und eine fast unbegrenzte Möglichkeit von Satzkompositionen zur Vefügung.

    Dies spiegelt das menschliche Denken, welches sowohl einen logischen Gehalt besitzt als auch emotionale und affektive Färbungen annimmt.
    Der Satz (Zitat):“Wenn A und B ein gemeinsames kulturelles Allgemeinwissen haben, dann kann B die Information von A verstehen.” stimmt nur teilweise. Oft versteht man sich auf einer Ebene, nicht aber auf einer anderen und viele sprachliche Äusserungen enthalten oft mehrere Ebenen. Selbst die Kommunikation nur über den Sprachkanal ist also bereits mehrkanalig.

    Menschliches Denken und Fühlen in einem technischen System nachzubilden ist äusserst schwierig, weil ein grosser Anteil davon nicht explizit benannt und bekannt ist und Inhalte über ein Netz von Assoziationen verbunden sind. Die grössten Aussichten auf ein technisches System, das denkt wie der Mensch hat darum wohl eine Nachbildung des menschlichen Gehirns – und selbst bei diesem Ansatz muss man noch die Hürde der “Erziehung” des Hirns überwinden, denn wir sind keine leeren Seiten sondern – um im Bild zu bleiben – vollbeschrieben und bekritzelt wie gewisse Wände und Bücher.

  69. @Blume

    Zitat Blume:
    “Mir stellt sich aber eine Frage: Unsere Form der Kommunikation über das Internet, dortselbst wiederum vermittelt über Symbole wie Zahlen und Buchstaben, wäre noch unseren Vorfahren vor wenigen Jahrzehnten als gröbster Verstoß gegen physikalische Regeln erschienen, oder? Wir sind nicht in einem Raum, sind uns nie (zumindest mir nicht bewusst) begegnet, haben uns auch nie gehört – und tauschen dennoch Geistiges (!), nämlich Argumente und Fragen aus. Teilhard de Chardin hätte das wohl Noosphäre genannt, mich würde aber wirklich interessieren, wie Online-Kommunikation in Ihr (m.E. sehr interessantes!) Paradigma passt.”

    Ich glaube, hier sollte man zunächst klären, in welchem Verhältnis Wissenschaft (Epistemologie) zur Realität (Ontik) steht: Wir beobachten die Realität (wobei natürlich viele Beobachtungen davon beeinflusst werden, was wir bereits wissen) und versuchen, uns darauf einen Reim zu machen, also Theorien zu entwickeln. Diese Theorien sollen die bereits gemachten Beobachtungen erklären und Vorhersagen für weitere machen. Eine solche Theorie, die aus Beobachtungen gewonnen wurde, war Newtons Gravitationstheorie. Sie konnte übergreifend sowohl den Fall eines Apfels Richtung Erde als auch die Bewegung der Planeten um die Sonne erklären. Später hat man dann weitere Beobachtungen gemacht, z.B. die Merkurbahn, die sich mit der Newtonschen Theorie schlecht erklären lies und heute am besten mit der Einsteinchen ART beschreibbar ist. In der Zukunft machen wir vielleicht weitere Beobachtungen, die wieder neue Theorien erfordern. An den bereits gemachten Beobachtungen ändern diese neuen Theorien aber nichts, sie beschreiben sie nur immer besser.

    Das Grundprinzip sollte klar sein: Nicht die Physik gibt die Regeln vor, sondern Theorien sind Destillate unserer Beobachtungen. Wenn Theorien im Widerspruch zu den Beobachtungen stehen (oder auch zueinander widersprüchlich sind), ist das ein Problem der Theorien, nicht der damit beschriebenen Realität.

    Jetzt können wir zurück zum Physikalismus, Emergentismus und Panpsychismus am Beispiel der Atome. Mit den Atommodellen in der Physik können wir bestimmte Beobachtungen der Realität für sehr kleine Objekte sehr gut beschreiben. Diese Beschreibungen der Realität durch die Physik waren und sind so erfolgreich, dass wir damit unsere Welt verändern können, eben z.B. das Internet schaffen.

    Der Physikalismus behauptet nun, dass die in der Atomwelt gemachten Beobachtungen bzw. genauer die daraus abgeleiteten Theorien *hinreichend* dafür sind, auch beliebig komplexe Phänomene gut zu beschreiben. Der Emergentismus und der Panpsychismus sind anderer Meinung, aber auf eine unterschiedliche Weise.

    Der Panpsychismus ist der Meinung, Atome müssten irgendwelche Eigenschaften haben, die Geistiges hervorbringen. Hier wird bereits stillschweigend angenommen, Atome wären etwas Reelles und man könne nur nicht alle ihre Eigenschaften mit den Methoden der Physik untersuchen. Der Emergentismus ist hingegen der Meinung, dass Atome nur Modelle sind, die uns aufgrund ihrer Methodik genau das über die Realität zeigen, was wir auf dieser Beschreibungsebene eben erhalten können: Messbare Eigenschaften wie Masse, Ladung, Spin und noch einiges mehr, was wir immer in gewissen Verbünden finden und deshalb konzeptionell zu “Teilchen” zusammenfassen, die bestimmte Wertekombinationen dieser physikalischen Eigenschaften haben.

    Der Emergentismus nimmt an, dass man geistige Eigenschaften auf der Ebene, auf der man mit Atomen arbeitet, nicht finden kann, weil das Atom(modell) eben kein Bewusstsein hat und keinen freien Willen usw. Um diese Eigenschaften zu untersuchen, braucht man komplexere Systeme, z.B. Menschen. Was von der Physik gefordert wird, ist lediglich (und eine Selbstverständlichkeit), dass die beobachteten geistigen Eigenschaften sich nicht im Widerspruch zu den Beobachtungen der anderen Wissenschaften(, auch der Physik,) befinden dürfen.

    Oder mit anderen Worten: Für den Panpsychismus sind Atome *hinreichend*, um Geist zu erklären. Aber dabei muss der Panpsychist Eigenschaften der Atome annehmen, die in der Physik nicht gemessen werden können. Für den Emergentismus sind Atome und ihre Eigenschaften lediglich *notwendig*, die Wechselwirkungen, die den Geist ausmachen, lassen sich aber nicht auf Atomebene untersuchen, weil sie dafür zu komplex sind. Als Vergleichsliste die drei Auffassungen:

    – Der Physikalismus betrachtet Atome als hinreichend, um alle Phänomene zu beschreiben. Bewusstsein und freier Wille sind Schein- oder Epiphänomene.
    – Der Panpsychismus hält Atome für hinreichend, unterstellt ihnen aber geistige Eigenschaften, die physikalisch nicht beobachtbar sind.
    – Der Emergentismus betrachtet Atome als Modelle, die zu einfach sind, um die komplexen Wechselwirkungen zu beschreiben, die z.B. das Bewusstsein ausmachen.

    Und um abschließend auf das Internet und seine Kommunikationsmöglichkeiten zurückzukommen: Das ist *möglich*, weil es innerhalb der Grenzen, die die Realität uns auf physikalischer Ebene setzt, realisiert werden kann. Aber es existiert, weil wir das *wollen* und für *sinnvoll* halten. Und diese beiden Begriffe “Wille” und “Sinn” lassen sich eben nicht mit physikalischen Mitteln beschreiben. Atome wollen nichts und erkennen auch keinen Sinn in dem, was sie tun.

  70. @ Joker

    e)
    Die Aussage, dass das Geistige dort auch wirkt, nicht nur “da ist” – wirkt insofern, als es Zustände der Intentionalität, gewisse Zwecke und qualitative Eigenschaften in die Welt (des Erlebens) bringt.

    j)
    Es gibt Geistiges – als Eigenschaften “sui generis”, aber niemals ohne, also losgelöst von Materie.

    k)
    Im weiteren Sinne handelt es sich um einen Eigenschaftsdualismus – sofern wir den alten Dualismus von Descartes akzeptieren, also eine “entleerte” Auffassung von Materie.

    o)
    ontogenetisch, als “qualitativer Sprung”, z.B. John Searle behauptet das.

    r)
    Ich glaube, dass der Mensch einfach das Bedürfnis hat, ein in sich schlüssiges Weltbild zu entwickeln. Der Panpsychismus ist dabei ein Versuch von vielen, die Welt um uns herum ein Stückchen verständlicher zu machen …

  71. @Köppnick @Blume 05.12.2012, 17:29

    Der Kern des obigen Kommentars ist:
    Alle Theorien, die nicht in Beobachtungen verankert sind, sind Spekulationen.

    Der Panpsychismus, aber auch die Stringtheorie sind solche Spekulationen. Wobei bei der Stringtheorie bestätigende oder widerlegende Beobachungen mindestens noch in Reichweite sind. Beim Panpsychismus vermisse ich hingegen irgend einen Hinweis wie man seine Grundlagen und seine Aussagen überprüfen kann.

    P.Späth motiviert den Panpsychismus sehr gut, wenn er schreibt:
    “Ich glaube, dass der Mensch einfach das Bedürfnis hat, ein in sich schlüssiges Weltbild zu entwickeln. Der Panpsychismus ist dabei ein Versuch von vielen, die Welt um uns herum ein Stückchen verständlicher zu machen …”

    Doch für mich sind Bedürfnisse nicht ausreichend für Welterklärungen, ich wil der Wahrheit näherkommen. Der Panpsychismus stellt darauf ab, dass er für viele Menschen mit einem gewissen (vor-naturwissenschaftlichen) philosophischen Hintergrund plausibel ist.

  72. @maxim

    “Der erste Satz stimmt einfach nicht. Eine Rückkopplung auf die eigenen Strukturen und Aufteilen des KNNs in mehrere Bereiche ist zum Beispiel ein Gegenbeispiel.”

    Inwiefern ist das ein Gegenbeispiel? Wird das neuronale Netz erregt und geht damit in den Zustand A, während dieser über den Zustand B und irgendeine Rückkopplung den Zustand C erreicht.
    Wie kann es damit einen semantischen Gehalt erfassen?

    Ein neuronales Netz braucht immer einen Beobachter, der die Muster auch interpretiert.

    Letztenendes kann man das Gehirn auch nur als System verstehen, welches Input verarbeitet und am Ende ggf. einen Output über bspw. Motorneuronen ausgibt.

  73. Neben der Motivation steht auch die logisch-metaphysische Argumentation, deren Kernthesen ich (auf Seite 115 des Buchs) wie folgt beschreiben würde:

    (1) Es gibt materielle Eigenschaften.
    (2) Es gibt geistige Eigenschaften.
    (3) Geistige Eigenschaften sind nicht identisch mit materiellen Eigenschaften.
    (4) Geistige Eigenschaften sind keine emergenten Produkte der Materie.

    (S) Die gesamte Materie und das gesamte Universum haben geistige
    Eigenschaften.

    Die Kernthesen lassen sich m.E. gewinnen durch eine Kritik an den physikalistischen Lösungsansätzen und durch weitere Überlegungen zum Aufbau des Kosmos und der Lebewesen sowie der Evolution.

  74. @Holzherr

    Ihr letzter Kommentar bringt es auf den Punkt: Wissenschaftliche Theorien müssen immer auf Beobachtungen aufbauen, sonst verletzt man das Paradigma der Wissenschaftlichkeit. Man kann aber daneben durchaus Metaphysik betreiben, zumal die Wissenschaft an ihren Grenzen auch von Spekulationen vorangetrieben wird (“Was wäre, wenn?”).

    Es gibt auch Bereiche der Realität, die sich nach derzeitigem Stand eh nicht wissenschaftlich untersuchen lassen, dazu zählen der Natur der Sache nach z.B. einmalige und nicht wiederholbare Ereignisse. Theorien über sie wären nicht falsifizierbar. Ob z.B. unser phänomenales Erleben (die Qualia) dazu zählen, darüber lässt sich trefflich streiten, aber derzeit nicht entscheiden.

  75. @P. Spät: “Es taucht hier öfters der Kritikpunkt auf, dass der Panpsychismus dieses und jenes nicht beweisen könne. Streng wissenschaftlich beweisen kann keine philosophische Theorie etwas – genau deshalb ist Philosophie METAphysisch und existiert als (leider oft allzu abkapppselte) Disziplin neben den Naturwissenschaften. Philosophen können lediglich argumentieren und versuchen, die Wirklichkeit in ihrer GANZEN Fülle zu deuten. Und Theorien, die sich selbst als wasserdicht oder bewiesen darstellen, halte ich für verdächtig wenn nicht gar dogmatisch.”

    Unsinn. Keine Theorie, auch keine naturwissenschaftliche kann jemals etwas beweisen. Popper lesen. Aber jede Theorie die empirische Aussagen tätigt (Geist auf Atomebene ist eine empirische Aussage) unterliegt dem Falsifikationsprinzip und muss sich daher überprüfbar machen. Das gilt daher natürlich auch für den Panpsychismus, der genau solche Aussagen tätigt. Die Ausrede: ‘Können wir nicht fassen, weil auf Planck-Länge’, ist nicht anderes, als die Ausrede ‘ Gott können wir nicht fassen, weil er außerhalb unseres Erfahrungsbereich liegt’. Unangreifbar wollen sie sich machen. Ihr Panpsychismus ist daher nicht nur eine metaphysische Theorie, sondern durch die Immunisierung mittels dem Schwurbel-Begriff “Planck-Länge” (erklären sie Philosoph mir Physiker mal was sie darunter verstehen, bin schon gespannt) dogmatisiert sich diese Theorie noch.

    Dass ich einen Philosophen zur Einführung einen Wiki-Link anbieten muss, ist die eigentliche Schande. Dass das auf einen Wissenschaftsblog erscheint, ist in der Tat eine Frechheit. http://de.wikipedia.org/…Immunisierungsstrategie
    Oder mal bei Hume und Abgrenzungs

  76. @ Gustav

    Sie schreiben:
    “Aber jede Theorie die empirische Aussagen tätigt (Geist auf Atomebene ist eine empirische Aussage) unterliegt dem Falsifikationsprinzip und muss sich daher überprüfbar machen.”

    Ich bezweifle, dass es sich hierbei um eine epmpirische Aussage im strengen Sinne handelt, da schon der Geist meines menschlichen Gegenübers empirisch nicht im Labor greifbar ist. (Stichwörter: Problem des Fremdpsychischen, Wittgenstein, Qualia-Debatte). Das ist ja gerade der Clou des Leib-Seele-Problems, dass es sich um Fragen dreht, die nicht rein empirisch belegbar sind.

    Wenn der eliminative Physikalist behauptet, dass es keine geistigen Eigenschaften gibt, frage ich mich: Wo ist der empirische Beleg? Und füge im gleichen Atemzug hinzu: Ich süpüre diese Eigenschaften tagtäglich. Wenn der nicht-reduktive Physikalist (= Emergenztheorie) behauptet, dass geistige Eigenschaften aus der “nicht-geistigen” Materie plötzlich hervorgehen, fage ich mich: Wo ist der empirische Beleg? Und wie soll das funktionieren?

    Und: Was wäre Ihre Lösung des Leib-Seele-Problems????

    Ich kenne die Grenze de Planck-Skala so, dass wir unterhalb dieser Grenze keine fundamentaleren Abstände mehr
    feststellen können, da wir bei einer empirischen Messung derart viel Energie aufwenden müssten (oder einer Sonde zuführen), dass ein winziges Schwarzes Loch entstehen würde – und dieses Schwarze Loch würde alle Informationen in
    sich aufsaugen, so dass uns der Blick auf diese Abstände verwehrt wird. Bitte korrigieren Sie mich, wenn ich daneben liege!

  77. @Gustav

    Anderen schon das Infragestellen untersagen und gleich die Wissenschaftlichkeit absprechen zu wollen ist ebenfalls eine beliebte Immunisierungsstrategie.

    Was man auch immer über den Panpsychismus denken mag, so legt er m.E. doch den Finger in eine Wunde: Auf die Frage, ob Geistiges in der Materie von Anfang an präsent ist oder später (woher? wie?) hinzukommt, vermag der Physikalismus (bislang?) nicht zu antworten.

    Dafür das Problembewusstsein zu schärfen, über häufig verwendete Begriffe neu nachzudenken und ggf. zu neuen, empirischen Fragestellungen vorzustoßen – das kann m.E. durchaus ein Ertrag philosophischer Debatten sein.

    Solange wir nicht alle Antworten haben, sollten uns (An-)Fragen nicht ärgern, sondern anspornen.

  78. @Michael

    Ich denke, du missverstehst einen Großteil der Gegenstimmen hier. Es haben wohl nur wenige naturwissenschaftlich denkende Menschen etwas gegen andere Denkansätze. Was an der Einstellung von Herrn Spät und anderen “Post-Physikalisten” stört, ist dass sie den Vertretern anderer Theorien unterstellen, nicht ausreichend nachgedacht zu haben.

    Ein wenig von der Demut, die du von anderen einforderst, stünde auch dem Panpsychismus gut zu Gesicht.

  79. @Joachim: Hard vs. Soft Sciences

    Da bin ich ganz bei Dir. Wenn hier nur Jubelarien auf den Panpsychismus gesungen würden, würde ich in der Tat die kritischen Stimmen stärker machen.

    Irgendwann habe ich mich damit abgefunden, als Evolutionär notwendig zwischen den Wissenschaftskulturen zu stehen: In der Religionswissenschaft rümpfen immer noch einige die Nase darüber, wie man sich nur so intensiv mit den “Reduktionisten” aus Physik, Chemie oder Biologie einlassen könne – von Kultur und Geist hätten die doch keine Ahnung und der “Evolutionismus” sei doch irgendwie schon mal gewesen und “überwunden”. Und auf Twitter und in der Blogosphäre dominieren physikalistische Stimmen, die Soziologen, Ethnologinnen oder gar Philosophien schnell als “Geschwätz” abtun. (Das Ganze übrigens interessanterweise auch mit je passendem Gender-Gap.)

    Klar könnte ich es wärmer haben, wenn ich mich einfach immer auf eine Seite schlüge und damit Schulterklopfen von Gleichgesinnten einheimste. Aber der Reiz des Bloggens liegt für mich genau darin, von den unterschiedlichsten Perspektiven zu lernen. So haben mich hier beispielsweise @Maxim und @Köppnick bereits auf ganz neue (Denk- und Lese-)Spuren gebracht, ein pfiffiger Twitterer auf das neue “Romantic Reductionist” von Christof Koch hingewiesen usw. Für diese Debatten & Chancen liebe ich das Netz und die scilogs.

  80. Der Geist und die nackte Materie

    Eine wichtige Motivation des Panpsychismus wie er hier dargestellt wird, scheint mir der Wunsch der Verfechter solcher und verwandter Konstrukte, dass die Welt irgendwie belebt, ja beseelt ist. Chrys erwähnt in seinem Kommentar die Einschätzung des Mind-Theoretikers Douglas Hofstadter, dass jene Vorstellungen, die dem Geistigen eine eigene Form von Stofflichkeit, ein eigenes Substrat, zuerkennen wollen, eine Art élan mental am Werk sehen, analog zum élan vital. In der Tat vertritt beispielsweise der scilogs-Autor Stehphan Schleim sowohl einen élan mental als auch einen élan vital.

    Auch Patrick Spät will uns offensichtlich eine Welt anbieten, die beseelt ist, was sich im folgenden Satz aus dem obigen Interview äussert:
    “Für ihn [den Physikalisten und Emergentisten] gibt es am Fundament der Welt nur nackte Materie, die wir mit unseren Formeln lückenlos beschreiben können.
    Eine entzauberte Welt also, in der auch wir Menschen nur aus seelenlosen Atomen bestehen und uns in nichts von einem Baum oder dem Inhalt eines Abfallkübels unterscheiden.
    Es geht Patrick Späth und anderen Geistsubstanzialisten/-vitalisten letztlich darum, den Narzissmus des Menschen zu bedienen. Diese Selbstliebe, dieser Glaube an die eigene Bedeutung des Subjekts, die dem Subjekt und allem was es wahrnimmt und über was es nachdenkt, überhaupt erst einen Sinn gibt, gilt es zu retten und zu bewahren.

    Die Ablehnung des Emergenzgedankens in diesen Kreisen hat ebenfalls psychologische und kulturelle Gründe: Ein Auto bleibt für uns letztlich ein Haufen Blech und allenfalls kann die Besprengung eines frisch eingeweihten Schiffes mit Weihwasser oder das Einhauchen von Lebensatem in eine Puppe diese zum Leben erwecken.

    Dabei hat der Emergenzgedanke, also die Idee, dass das Zusammenspiel von Elementen etwas völlig neues schafft, was in den Einzelementen noch nicht zu erkennen ist, vieles für sich, vor allem weil es eine Unzahl von überwältigenden Beispielen dafür gibt. Wie im Wikipedia-Artikel darüber erwähnt wird, brauchen wir nicht einmal zwischen schwachem und starken Emergentismus zu unterscheiden, denn auch wenn wir die Phyisk hinter dem Emergierenden verstehen (schwacher Emergentismus) kann das Resultat eine Welt mit ihren eigenen Gesetzen sein.
    Nehmen wir als Beispiel das Universum als Ansammlung von Galaxien mit Sternen unterschiedlicher Grössenklasse, mit Planeten von denen einige habitabel sind und so aussehen wie unsere Erde. Die Evolution eines solchen Universums wurde bereits im Rechner simuliert, wobei nur physikalische Grundgesetze zur Anwendung kamen. Und die entstandene Welt unterschied sich prima vista nicht von dem was hier heute durchs Teleskop beobachten.

    Was ist so gesehen die Funktion von Leuten wie Patrick Späth? Sie wollen die nackte Wahrheit einkleiden und unserer Welt einen Sinn geben und diesen Sinn gegen eine mögliche rein materielle Sicht verteidigen.
    Das erinnert mich an die japanischen Geishas, die gelernt haben, sich auf hohem geistigen Niveau mit ihren Kunden über Kunst, das Leben und alles was dazu gehört zu unterhalten und die zugleich wissen, dass es auch noch anderes als diese hohe geistige Welt gibt, dass dieses andere aber nicht direkt angesprochen werden soll, denn es wäre eine niedrige und das Ego des Kunden beleidigende Sicht, die Welt des Geistes nur als Schein zu betrachten, hinter der sich die schnöde Materialität verbirgt.

  81. Wo gibt es denn wirklich Emergenz in der Natur? Selbst Temperatur und Druck, die klassischen Beispiele, lassen sich problemlos auf molekulare “Bewegungen” zurückführen. Da entsteht nichts Neues, nur weil wir etwas aus der makroskopischen Ebene heraus betrachten.

  82. @ Martin Holzherr

    Danke für Ihren Kommentar.
    An einigen wichtigen Punkten muss ich widersprechen:
    1.) Ich bin kein Vitalist – und argumentiere in meinem Buch entschieden gegen den Vitalismus. Lebewesen sind belebt, nicht Atome oder das Fundament der Welt.
    2.) Der Begriff der Seele spielt für den graduellen Panpsychismus keine Rolle … Seele verstanden als (unzerstörbarer, unsterblicher, gottgegebener) Persönlichkeitskern, oder als eigenständige Entität.
    3. Der (graduelle) Panpsychismus trägt m.E. eher dazu bei, dem Narzzismus des Menschen entgegenzuwirken: Der Mensch steht in einer langen Kette der Evolution und ist ein Lebewesen unter vielen, bei dem sich das Geistige in besonderer Weise entfaltet hat. Teilhard de Chardin wurde nebenbei bemerkt nicht umsonst ins chinesische Exil verbannt, auf den Index gesetzt und mit einem lebenslangen Publikationsverbot belohnt.

  83. @ PeterK

    Ja, in der Tat: natura non facit saltus, ich hab jedenfalls noch keinen gesehen und von keinem gehört … und deshalb glaube ich weiterhin, dass eine plötzliche Eemrgenz geistiger Eigenschaften ihrerseits gegen alle uns bekannten Naturgesetze und Vorstellungen des Kosmos verstößt.

  84. @ Joachim

    Sie schreiben:
    “Ein wenig von der Demut, die du von anderen einforderst, stünde auch dem Panpsychismus gut zu Gesicht.”

    Hmm, ich habe mehrmals geschrieben, dass es sich um eine empirisch nicht belegbare, dafür auf logisch-metaphysischen Überlegungen beruhende These über den aufbau unserer Welt handelt – eine Theorie, die für Kritik, Ergänzungen, Korrekturen offen ist, da sie nur ein Gerüst darstellt. Und ich behaupte nicht, dass der Panpsychismus die Wahrheit mit Löffeln gegessen hat …
    aber dass ich diese These verteidigen möchte, liegt ja auf der Hand 🙂

  85. Sympathie und Antisympathie

    Als Kind habe ich mich immer sehr interessiert dazu gesetzt, wenn mein Vater dem internationalen Frühschoppen mit Werner Höfer versucht hat zu folgen.

    Es hat eine Weile gedauert, aber dann habe ich begriffen worum es in dieser systematischen Kommunikation geht (siehe oben). Teilnehmer und Zuschauer haben keine andere Chance ausser eine möglichst gute Show mit Rauchen, Saufen und Labern abzuliefern, bzw. als gutbürgerlich-gebildeter Bürger in Suppenkaspermentalität auf Sündenbocksuche den Stillstand zu geniessen – eine Spielwiese für Logopäden, Rethoriklehrer und andere Laberkünstler 🙂

    Wenn ihr den Geist in diesem menschenunwürdigem System auf Bewußtseinsentwicklung mittels zeitgeistlichem Verstand erforschen wollt, dann werden euch keine noch so sensiblen Sensoren an Nervenenden und … eine allseits befriedigende Antwort bringen. Ihr müßt mit krassen Methoden die schon unsere Vorfahren kannten auf Reisen in eure konfusionierte Psyche gehen, auch wenn diese Methoden heute aus Gründen der Staatsräson verpönt und verboten sind.

    “… bei dem sich das Geistige in besonderer Weise entfaltet hat.”

    Und wenn wir erst einmal den zweiten zu erwartenden GEISTIGEN Evolutionssprung machen, dann wird sich deutlich zeigen was die garnicht Weise systemrationale BEWUßTSEINSBETÄUBUNG (der geistige Stillstand seit der “Vertreibung aus dem Paradies”) für besonders negative / dumme Auswirkungen entfaltet!

  86. @Patrick Spät

    Hallo Herr Dr. Spät,

    was ist für sie das “Geistige”? Will sagen, der pochende Finger? Bewusstsein (wobei die Frage, was dies nun ist, selbst jahrelange Debatten füllt)? Denke, wenn sie das noch näher erläutern könnten, wäre es leichter diese Diskussion zu führen. Sei es, dass das Geistige, dann vielleicht bereits erklärbar ist oder eben nicht und weiterer Untersuchung bedarf. Aber dazu braucht es eben eine eindeutige Beschreibung.

    Vorab besten Dank.

  87. @ boris blix

    @ boris blix:

    Danke! Vielleicht hilft es weiter, ich versuche es mal kurz und bündig:

    GEIST
    Das Geistige und geistige Eigenschaften beschränken sich nicht auf bewusste oder gar selbstbewusste Erlebnisse. So verstanden ist das Geistige grundlegender als das Bewusstsein, und umgekehrt ist das Bewusstsein nur die
    Spitze des (geistigen) Eisbergs. Knapp 99 Prozent der geistig-kongitiven Vorgänge des menschlichen Hirns laufen unbewusst ab. Der Geist ist in einem weiten Sinn die Möglichkeit, Informationen zu verarbeiten. Was ist eine Information?
    “A ‘bit’ of information is definable as a difference which makes a difference” (Gregory Batesons klassische Definition). Vergleichbar mit dem Aufbau eines Morsealphabets zeichnet sich die einfachste und damit rudimentärste Form einer Information durch binäre Befehlszeilen aus: Wie bei einem Münzwurf gibt es ein ‘oben’ oder ‘unten’, ein binäres System bestehend aus ‘ja’ oder ‘nein’. Hat eine Entität die Fähigkeit, solche Informationen zu verarbeiten, dann hat sie geistige Eigenschaften. (Vgl. auch die Definition von Edelman/Tononi weiter oben in den Kommentaren, wo es um Dioden etc. geht.) Der Begriff ‘Geist’ bezieht sich demnach sowohl auf das bewusste Erleben als auch auf nichtbewusste Prozesse der Informationsverarbeitung, wie sie z.B. schon bei Mikroorganismen zu beobachten sind.

    BEWUSSTSEIN:
    Eine Entität hat ein Bewusstsein, wenn sie etwas erlebend wahrnimmt – ganz gleich, wie stark oder schwach dieses Erleben ausfällt.

    SELBSTBEWUSSTSEIN:
    Impliziert (zusätzlich zum Bewusstsein), dass sich die erlebende Entität über ihre Bewusstheit bewusst ist. Sie kann ihre Bewusstheit reflexiv auf sich selbst zurückspiegeln und sich ihrer selbst gewahr werden.

  88. @Patrick Spät

    Gleichfalls Danke für die rasche Antwort:

    ja finde ich hilfreich, da ich nun besser einordnen kann, worüber wir diskutieren.

    zum GEIST
    Sie schreiben: “Der Geist ist in einem weiten Sinn die Möglichkeit, Informationen zu verarbeiten. Definitionen zum Begriff der Information hatten sie angefügt.

    Nun, kann es Information ohne materielle Entsprechung geben? Hm.
    Nehmen wir die Zahl eins.
    a) kann erscheinen in direkter materieller Ausprägung, aber das meinen wir ja nicht

    also

    b) In Gedanken. Kann es eins ohne elektrochemische Abbildung in Gehirnen (denke auch manche Gehirne ausserhalb des Menschen haben einen zumindest einfachen Begriff von Zahlen) geben? Ich zumindest weiss nicht wie.
    Dann wären Zahlen ggf. (nur) durch Materie und ihre Umwandlung abbildbar. Möglicherweise ist es mit Zeichen, Buchstaben, Worten, Sätzen, ebenso.
    Materie und ihre Umwandlung folgt (oft) erstaunlich einfachen Gesetzmässigkeiten. Komplexität entsteht aus der vielfachen Wiederholung einer einfachen Gesetzmässigkeit in der Regel über relativ lange Zeiträume.

    Damit ist nicht gesagt diese zugrunde liegenden “Gesetzmässigkeiten” einfach zu entdecken. Oft sieht man ja den sprichwörtlichen Wald vor lauter Bäumen nicht.

    Denke, diese (sehr grobe) Abfolge kann die Entstehung von Geist (Informationsverarbeitung) aus “toter Materie” verständlich machen. Damit sage ich nicht, dass es GENAU SO sein muss. Aktuell weiss ich nur nichts was plausibler wäre.

    Sie?

  89. @Spät

    Wenn wir mit “Geist” in einem erweiterten Sinn mit “Informationsverarbeitung” gleichsetzen, dann beginnt Ihr Geist tatsächlich schon im Unbelebten, denn das, was in der Quantenphysik passiert, ist auch in den Termini der Informationsverarbeitung beschreibbar – sonst könnten wir keine (Quanten)Computer bauen. Es gibt keine “abstrakte” Information, jede Information ist immer an einen materiellen Träger gekoppelt.

    Die philosophische Strömung, die das am klarsten (und dabei nichtdualistisch!) ausdrückt, ist der neutrale Monismus.

    Übrigens passt dieser auch am besten zu meinen eigenen Vorstellungen (und zum Emergentismus): Je nachdem, auf welcher Betrachtungsebene wir Fragen an die Natur stellen, erhalten wir unterschiedliche Antworten aus *derselbe*n Realität: In der Physik in der Notation (den Modellen) der Quantenphysik, in den Neurowissenschaften bzw. der Philosophie des Geistes in Form der Qualia.

  90. Ich denke, der Neurowissenschaftler Herr Roth hat es treffend formuliert, als er meinte, dass man das eigentliche Gehirn ja nicht wahrnehmen könnte.
    Wir können ja nur die Dinge wahrnehmen, welche wir durch unsere Sinnesorgane aufnehmen. Jede andere Wirklichkeit bleibt uns verborgen. Wir nehmen eben nur das Gehirn als Materiehaufen wahr.

    Ich denke es ist das Gleiche, wie wenn wir in einem Traum nach dem Sitz unseres Bewusstseins suchen. Wir nehmen nur das wahr, was schon der Inhalt unseres Bewusstseins ist. Das eigentliche Gehirn, welches träumt, ist nicht Teil der Erfahrungswelt des Traumes und lässt sich daher dort auch nicht finden.
    Ich denke das ist eine treffende Metapher.

  91. @ P. Spät: Alles springt

    “Da schiebt uns schon die Planck-Skala einen Riegel vor.”

    Wenn man über Planck-Länge und Planck-Zeit philosophiert, sollte man gleichzeitig darüber nachdenken, ob es nicht besser wäre, die Grundannahme “Natura non facit saltus” (Die Natur macht keine Sprünge) fallen zu lassen.

    Mein Latein ist etwas eingerostet, bitte übersetzen Sie: “Alles springt”

  92. @ P. Spät: Pappkamerad?

    Spät:
    “Der Emergentist glaubt, dass aus dem rein materiellen Hirn plötzlich geistige Eigenschaften “auftauchen””

    Joker:
    “Wie soll ich dieses “plötzlich” verstehen, phylogenetisch und/oder ontogenetisch? […]Welcher Emergentist glaubt das oder beschreibt das so”

    Spät:
    “ontogenetisch, als “qualitativer Sprung”, z.B. John Searle behauptet das.”

    Nun habe ich die Stelle bei Searle nicht auf Anhieb gefunden, die Sie im Sinn haben. Bei Searle könnte ich mir vorstellen, dass er den Begriff benutzt um z.B. die Differenz zwischen schlafender und wacher Person herauszuheben. So mag hier der Übergang zum Bewusstsein als ein “qualitativer Sprung” wahrgenommen und auch so bezeichnet werden. Der Vorgang des Aufwachens könnte dessen ungeachtet aber sehr wohl als ein kontinuierlicher Prozess verstanden werden (Planck mal außer Acht gelassen).

    Ich kann mir nicht vorstellen, dass Searle mit seiner Auffassung eines “biologischen Naturalismus” irgend einen Zeitpunkt in der Ontogenese eines Menschen vermutet, ab dem “plötzlich” Geist auftaucht. Aber vielleicht können Sie ja noch etwas Genaueres dazu sagen.

    Je nach Emergenz-Theorie (z.B. starker oder schwacher), könnten zwar nicht zum Verschwinden bringende Erklärungslücken vorhanden sein oder auch nicht, ich wage hier aber einmal die Behauptung, kein Emergentist ist auf “plötzliche” oder sprunghafte Änderungen eines Systems festgelegt, im Sinne von “plopp”.

    Rhetorische Frage: Wieso kommt mir gerade jetzt wieder der Begriff “Pappkamerad” in den Sinn?

  93. Versuch von Antworten

    @ Joker:
    Hmm, schwierig. Ich glaube, dass materielle und geistige Eigenschaften stets zusammenspielen müssen – und die einen nicht ohne die anderen bestehen können. Der Bereich der Mathematik ist allerdings seit jeher tricky: “Gibt” es Zahlen? Wenn ja, wo?

    Mein Latein ist auch eingerostet 🙂 Aber das, was da springt, macht m.E. keine qualitativen Sprünge, sondern solche, bei denen wir alles (zumindest theoretisch) nachvollziehen können.

    Searle sieht geistige Eigenschaften “sui generis”, spricht ihnen also Eigenschaften zu, die der (klassischen) Definition von Materie fehlen (Qualität, Intentionalität, …). Er geht dabei – so lese ich ihn mit vielen anderen auch – von qualitativen Sprüngen aus, also einer starken Emergenz. Auch wenn er das Bewusstsein als einen Vorgang sieht, der wie die Verdauung einfach biologisch vor sich geht – den Sprung muss er schlucken, da er der Materie sämtliche Eigenschaften abspricht, die er dem Bewusstsein vehement zuschreibt.

    @ Köppnick:
    Sie Schreiben:
    “Wenn wir mit “Geist” in einem erweiterten Sinn mit “Informationsverarbeitung” gleichsetzen, dann beginnt Ihr Geist tatsächlich schon im Unbelebten”
    Ja, genau davon geht der Panpsychismus aus! Und der Neutrale Monismus ist aufs engste mit dem Panpsychismus verwandt, ja, bei manchen Denkern überschneiden sich diese Theorien stark (W. James, H. Feigl, B. Russell … D. Skrbina geht sogar davon aus, dass jede Form des Neutralen Monismus in einen Panpsychismus mündet).

  94. Diagnose Unwissen durch Dogma ersetzen

    Panpsychismus macht Setzungen/Aussagen dort wo wir noch wenig wissen. Damit erfüllt er ein uraltes Bedürfnis von Menschen, das in allen mir bekannten Kulturen zu beobachten ist: Der Mensch will in einer bekannten Welt leben. Er begnügte sich noch nie mit der Aussage: Wir wissen nicht wer wir sind, woher wir kommen und wohin wir gehen

    Heute wissen wir schon viel mehr darüber, aber immer noch nicht alles – und wir werden nie alles darüber wissen. Sich damit abzufinden, sich damit vorerst einmal zu begnügen, das genau unterscheidet denjenigen, der mit Unvollständigkeit und offenen Perspektiven leben kann, von demjenigen der Antworten braucht und sich mit Antworten begnügt, die nur etwas behaupten aber nichts wirklich erklären.

  95. @Chrys

    Vielen Dank für den Link zu dem spannenden Video!

    Mir fiel dazu – gerade auch im Hinblick auf die Mandelbrotmengen – auch dieses Video von Salman Khan (Khanacademy) ein, der sich ausgehend von den gleichen Grundüberlegungen der Ablehnung von Intelligent Design und der Vereinbarkeit von Evolution und Gottesglauben widmet (2. Video hier):
    http://www.scilogs.eu/…n-and-evolutionary-theism

    Ein “Video-Dialog”, wie cool! 🙂

  96. “… und die einen nicht ohne die anderen bestehen können.”

    Mensch bedeutet ALLE. Wenn also “Individualbewußtsein” glaubt und …, dann bewegt / spaltet es sich im Rahmen des Schicksals der kosmischen Ordnung des Zentralbewußtseins (relativ leicht vorhersehbar), oder religiös ausgedrückt: eskaliert / kulminiert kreislaufend in der “göttlichen” Sicherung vor den Möglichkeiten des Freien Willens bei wirklich-wahrhaftigem Verstand von universeller Vernunftbegabung.

    Wenn Mensch sich nun als Mensch / ALLE im “gottgefälligen” Selbst- und Massenbewußtsein bewegt, dann ist die Materie SICHER keine illusionäre Krücke mehr, so daß Raum und Zeit OHNE … er-/gelebt werden können.

    Dieses setzt ALLERDINGS einen allumfassenden Sinn und EBEN NICHT die zufällige Einmaligkeit durch / in ein “PHÄNOMENAL” materielles Ereignis voraus!!!

    “Was man nicht im Bett tun kann, ist nicht wert, getan zu werden.” Groucho Marx 🙂

  97. kein

    “Zunächst glaube ich allerdings NICHT, dass der Panpsychismus in irgendeiner Weise die Erkenntnisse der Naturwissenschaft berrührt, verletzt oder außer Kraft setzt!”

    Genau das wollte ich mit dem Gedankenexperiment eigentlich nahelegen.

    “In welche Richtung zielen Ihre postulierten Zwecke ab?”

    Ich wollte im Grunde genommen zeigen: Wenn es Zwecke in der Natur gibt, dann können wir uns diesen nicht mit den “normalen” wissenschaftlichen Methoden nähern. Also nicht durch Experimente. Nur durch Beobachtung. Weil jedes Experiment vom Zweck des “normalablaufs” abweicht und eher den Zweck zeigt, denn die Menschen augenblicklich verfolgen. Es enthält damit ja schon eine psychologische “Aufladung”.

  98. graduellen Panpsychismus

    “Spät geht davon aus, dass in der Wirklichkeit eine gestufte Form von geistigen Eigenschaften anzutreffen ist: Nur dann, wenn ein Ding (eine Entität) in materieller Hinsicht ausreichend komplex ist, kann auch die korrespondierende geistige Seite komplexe Züge annehmen.” Quelle: Wikipedia

    Wie jetzt? Ist damit unsere Bildung, und / oder doch unser HIRN ALLEIN durch genetische / evolutionäre Entwicklung gemeint? Oder hat Wikipedia etwas völlig falsch verstanden und …???

    Wir sind alle im SELBEN Maß durchströmt vom Geist der “Gott” ist.
    Nur der Stand in unserer (ein)gebildeten Hierarchie von und zu materialistischer “Absicherung” (profit- und konsumautistische Bewußtseinsbetäubung) bestimmt den GRAD der Unterdrückung und Ausbeutung DIESER dann GLEICHHEIT!!!

  99. @ Wegdenker @ Horst

    @ Wegdenker:
    Pardon, das habe ich etwas krumm formuliert – und Sie haben natürlich recht: Der Panpsychismus verletzt nicht die naturwissenschaftlichen Gesetze, aber er geht natürlich über sie hinaus – auch bei der Frage der Zwecke (in der Natur).

    @ Horst:
    Die Formulierung bei Wikipedia ist korrekt: Nicht unsere Bildung ist gemeint, sondern unser Hirn – oder richtiger: unser gesamter Organismus (Leib) und der anderer Lebewesen.

  100. @ P. Spät: Panta ???

    Jetzt weiß ich auch wieder warum ich von Searle so wenig gelesen habe. Bei der Diskussion über die Möglichkeit von Künstlicher Intelligenz spricht er von “kausaler Kraft”, die Denken hervorbringen kann, und die biologische Materialien hätten, aber Siliziumchips eben nicht [1]. Er erklärt leider nirgends wie diese “kausale Kraft” konkret entsteht oder aus was sie entspringen könnte. Hier ist also zugegebenermaßen auch für mich, ein Sprung in seiner Theorie zu erkennen. Für mich klingt das Alles allerdings weniger nach einer Emergenztheorie des Geistes als vielmehr nach einer Erklärungslücke (genau an der Stelle, wo es eigentlich darauf ankäme), einer Schöpfung aus dem Nichts oder sogar einer gewissen Widersprüchlichkeit die hier auftaucht. Nun gut, es geht hier ja auch nicht um die Exegese Searlscher Texte sondern um Emergenz und den Panpsychismus.

    Sie sprachen von “Der Emergentist”, meinten also alle; ich sprach von “kein Emergentist”; vielleicht können wir uns in der Mitte einigen: Es gibt brauchbare, schlüssige Emergenztheorien des Geistes, die “qualitative Sprünge” auf der Wahrnehmungsebene zu erklären versuchen, die aber ohne “Sprünge im Naturgeschehen” auskommen.

    “Ich glaube, dass materielle und geistige Eigenschaften stets zusammenspielen müssen – und die einen nicht ohne die anderen bestehen können.”

    Ist der Panpsychismus etwa selbst eine Emergenztheorie, kann man beim Panpsychismus von einer synchronen Emergenz sprechen? Das Materielle gewinnt (graduell) an Komplexität, die sich eventuell sogar messen lässt, während das Geistige den unberechenbaren Anteil am Ganzen bezeichnet und sich synchron mitentwickelt – sozusagen als Trittbrettfahrer.

    Irgendwie entkommt keiner dem Emergenzbegriff. Wobei klar ist, dass Emergenz für sich alleine genommen gar nichts erklärt.

    [1] Geist, Gehirn, Programm, John Searle
    http://philosci24.unibe.ch/…te/searle/searle.pdf

    PS. Etwas peinlich: Nach vielen unbefriedigenden Versuchen meine Sentenz “Alles springt” von Online-Übersetzungshilfen ins Lateinische transferiert zu bekommen, ist mir nun klar geworden, ich hätte sie eigentlich viel lieber ins Griechische übersetzt (vielleicht können Sie mir ja damit helfen). Unbefriedigende Versuche waren es, denn ich wollte mich ja an eines meiner Lieblingsmottos anlehnen, und das stammt ja wohl aus dem Griechischen, nicht aus dem Lateinischen (wie ich erst spät bemerkte), “panta rhei”.

  101. @P. Spät

    Deshalb glaube ich, dass der Panpsychismus der Emergenztheorie einen entscheidenden Schritt voraus ist. Denn “aus Nichts kommt Nichts”

    (In Ihrem ersten Kommentar, Antwort auf Joachim)

    Auf diese Weise könnte man auch die Dreieckigkeit geometrischer Punkte beweisen: Es gibt Dreiecke, die aus solchen Punkten bestehen. Da aus Nichts Nichts kommt kann deren Dreieckigkeit nicht aus dem Nichts entstanden sein. Somit muß sie schon in den einzelnen Punkten angelegt sein, die somit schon eine Art simple, schwach ausgeprägte Dreieckigkeit besitzen.

    So ganz überzeugt bin ich davon noch nicht.

  102. Dr. Späts Geist

    Der Knackpunkt der panpsychistischen Theorie ist das Phänomen „Geist“, das dem gesamten Naturgeschehen untergeschoben wenn nicht gar zugrunde gelegt wird. Erstaunlich, dass erst nach Dutzenden Kommentaren jemand Dr. Spät danach fragt, was er mit diesem doch reichlich unklaren und mehrdeutigen Begriff überhaupt meint, wenn er ihn etwa auf Mikroorganismen und Atome anwendet. Seine Antwort besteht im wesentlichen in dem Satz „Der Geist ist in einem weiten Sinn die Möglichkeit, Informationen zu verarbeiten.“, also die berühmte obscurum-per-obscurius-Erklärung. Mein iPhone besitzt nach Dr. Spät also geistige Fähigkeiten.

    Da muss er sich schon den Einwand gefallen lassen, dass er in seinem Zwang zur akademischen Originalität nicht viel mehr als philosophische Gedankenspiele anbietet. Obwohl die Idee, Großbegriffe, wie Welt, Natur, Ordnung anders als physikalisch erklären zu wollen, sehr begrüßenswert ist, geht es doch auch immer um Details, die Differenzierung verlangen.

  103. @ David @ Jack Elsässer

    @ David:
    Hmm, ich denke, dass ihr Vergleich leider etwas hinkt, da sich Punkte, Dreiecke etc. allesamt im gleichen ontologischen Bereich befinden, während sich das Geistige und das Materielle in Ihrem Wesen unterscheiden – zumindest in einigen (bereits genannten) Aspekten.

    @ Jack Elsässer:
    Philosophie bietet immer (nur) Gedankenspiele, ich habe jedenfalls noch keine philosophische Theorie gesehen, die kein Gedankenspiel darstellt 😉

    Die Elementarteilchen Ihres iPhone besitzen simple geistige Eigenschaften, nicht aber Ihr iPhone als solches, also als Einheit. (Sprich, bei Ihrem iPhone ist das Ganze nicht wirklich mehr als die Summe der Teile, was das Geistige anbelangt, während Lebewesen eine Einheit bilden, vgl. Spinoza, Leibniz, Jonas und Whitehead, aber auch Systemtheoretiker wie Maturana oder Varela.)

  104. 99 Luftballons

    Das Titelbild von “Der Mensch lebt nicht vom Hirn allein”zeigt einen Luftballon. Das erinnert mich an Nenas Lied:
    Hast du etwas Zeit für mich,
    dann singe ich ein Lied für dich.
    Von 99 Luftballons
    auf ihrem Weg zum Horizont.
    Denkst du vielleicht g’rad an mich
    dann singe ich ein Lied für dich.
    Von 99 Luftballons
    und dass so was von so was kommt.

    Im Unterschied zum Buch beschäftigt sich Nena mit einem ernsten Problem aus der wirklichen Welt währen P.Späts Buch “nur” eine Anleitung gibt wie man die Welt interpretieren soll.

  105. @P. Spät

    “In einem gesunden Körper wohnt ein gesunder Geist” – etwa wieder so, obwohl das durch vieles widerlegt ist???

    Woher kommt die “Komplexität”, welches “Wunder” / “Phänomen” oder … bläht unser Hirn zur Fähigkeit von “geistigen Eigenschaften” – sind vielleicht die Mediclorianer daran schuld, weil wir ja nie konkret und selbstbewußt werden dürfen / können, damit das System auch weiter so flutscht wie es flutscht???

  106. @ Dr. Spät

    „Der Geist ist in einem weiten Sinn die Möglichkeit, Informationen zu verarbeiten“ – das nehmen Sie nun doch wieder zurück, obwohl ja diese Möglichkeit in mein iPhone tatsächlich schon eingebaut ist.
    Hab ich mir doch gleich gedacht, dass Sie das nicht so meinen wie sie es sagen. Wie Sie es ja nun auch ausdrücklich bestätigen, reden Philosophen ja sowieso nur unverbindliches Zeugs. Kant, Whitehead, Spät – alles nur Gedankenspiele. Die Wirklichkeit findet woanders statt.
    Zum Beispiel in den Elementarteilchen. Die haben ja viele Eigenschaften, einige sind physikalisch schon gut bekannt. Welche davon sind Ihrer Meinung nach geistig? Es wäre schön, wenn Sie das mal ganz konkret und ohne Metaphorik sagen könnten.

  107. @P. Spät

    “Der Panpsychismus verletzt nicht die naturwissenschaftlichen Gesetze, aber er geht natürlich über sie hinaus – auch bei der Frage der Zwecke (in der Natur).”

    Ja und nein. Denn der ‘Panpsychismus’ postuliert meines Erachtens Eigenschaften der Natur, die sehr wohl von Naturwissenschaftlern erforscht werden müssten, die aber der “normalen” Methodik nicht zugänglich sind.
    Man kann diese Eigenschaften aber nicht ignorieren, weil sie vielleicht relevant bei der Erklärung gewisser Phänomene sein könnten. (Mein Ameisenbeispiel ist da sehr gut. Man denke aber auch an das “Anthropologische Prinzip”!)

    “Hmm, ich denke, dass ihr Vergleich leider etwas hinkt, da sich Punkte, Dreiecke etc. allesamt im gleichen ontologischen Bereich befinden, während sich das Geistige und das Materielle in Ihrem Wesen unterscheiden – zumindest in einigen (bereits genannten) Aspekten.”

    Das halte ich für einen Petitio principii.
    WENN Geistiges und Materielles ontologisch verschieden sind, DANN muss man vielleicht eine “kausale Story” (“Urknall + physikalische Gesetze für Materie + Zeit = Zustand”) darüber erzählen, wie das Materielle in seinen gegenwärtigen Zustand gekommen ist und eine weitere “Story” darüber, wie das Geistige in seinen gegenwärtigen Zustand gelangen konnte.
    Und da scheint mir die Idee, dass der Geist quasi schon in kleinsten Teilchen gleichberechtigt mit der Materie existiert, schon zündend.

    @Martin Holzherr:
    Würden Sie diesen Vorwurf auch an Künstler, die an Schönen oder Wissesnchaftler, die an der Erklärung der Materie interessiert sind, erheben.
    Meines Erachtens hätten diese beiden Ideen schon Auswirkungen.

  108. Antworten

    @ Martin Holzherr:
    Sie schreiben:
    “Im Unterschied zum Buch beschäftigt sich Nena mit einem ernsten Problem aus der wirklichen Welt währen P.Späts Buch “nur” eine Anleitung gibt wie man die Welt interpretieren soll.”
    Noch schlimmer: Wie man sie interpretieren kann 😉

    @ Jack Elsässer:
    Ich würde – nicht um mich Popper-mäßig abzuschotten, sondern um der Logik willen – die elementarsten Teilchen unterschiedslos als Teilchen beschreiben, die geistige Eigenschaften haben. Die Frage ist also nicht, welches hat diese Eigenschaften und welche nicht, sondern welches ist das grundlegendste Teilchen/Ding/Welle/String etc., da sich aus diesem alles weitere aufbaut.

    @ Wegdenker:
    Ja, mit Ihrem “Ja und Nein” haben Sie es in diesem Fall bestimmt sehr gut getroffen: Es kommt auf die Frage an, die man klären möchte. Und bei manchen mischen sich zwangsläufig metaphysische Fragen unter.
    Und danke für Ihre Antwort an Herrn Holzherr. Da kann ich nur zustimmen.

  109. Dr. Späts “geistige Eigenschaften”

    Also, entweder haben Sie meine Frage nicht richtig verstanden, oder sie wollten sophistisch-gekonnt ausweichen, in dem Sie nun selbst eine andere Frage stellen, die sowieso niemand beantworten kann, nämlich welches das „grundlegendste Teilchen ist, aus dem sich alles weitere aufbaut.“
    Ausgehend von Ihrer Behauptung „Die Elementarteilchen Ihres iPhone besitzen simple geistige Eigenschaften“ hatte ich Sie ja nur gebeten, konkret zu sagen, welche der Eigenschaften eines Teilchen Sie für geistig halten. Das muss man doch erst mal wissen, ehe man Ihre Theorie versteht. Oder definieren Sie („kurz und bündig“, wie Sie es ja ursprünglich angekündigt hatten), was für Sie unter den Begriff „geistig“ fällt. Verständliche Beispiele würden es auch tun (aber wenn möglich keine Wortspiele).

  110. @ Jack Elsässer

    @ Jack Elsässer:

    Unter “geistig”, und den geistigen Eigenschaften, die Elementarteilchen aufweisen, verstehe ich ziemlich genau das, was ich am 04.12.2012 gepostet habe:

    “Die IITC („Information Integration Theory of Consciousness“) des Neurowissenschaftlers Giulio Tononi ist eine der wenigen neurowissenschaftlichen
    Theorien, die naturwissenschaftliche und phänomenologische (das, was wir tagtäglich erleben) Beobachtungen
    miteinander zu koppeln versucht. Tononis Theorie besagt, dass ein materielles Ding in dem Maße ein bewusstes Erleben aufweist, in dem es der Informationsverarbeitung fähig ist. […] Tononi arbeitet eng mit dem Mediziner und Nobelpreisträger Gerald Edelman zusammen. Die beiden wenden sich gegen alle neurowissenschaftlichen
    Theorien, die das Auftauchen des Bewusstseins auf einzelne Neuronen beschränken: Nicht eine Handvoll Neuronen sind für unser bewusstes Erleben verantwortlich, vielmehr bedarf es eines sogenannten „Kerngefüges“ („dynamic core“). Neuronen, die einem solchen „Kerngefüge“ angehören, gehen für kurze Zeiträume intensive Verbindungen ein. In den zeitlich beschränkten Kerngefügen werden Informationen verschiedener Neuronenverbände integriert und gebündelt. Erst dann, wenn bei dieser Bündelung ein bestimmter Schwellenwert der Komplexität erreicht ist, kann unser Bewusstsein hervorgehen. Wie bei einem Laser werden verschiedene lose Informationen konzentriert und so ihre Komplexität derart verstärkt, dass ein „Brennpunkt“ entsteht – beim Laser der energiereiche Lichtpunkt, bei Lebewesen derjenige Ausschnitt des Geistigen, den wir bewusst erleben. […]

    Die für einen Neurowissenschaftler – zumal für einen weltweit anerkannten wie Giulio Tononi – recht ungewöhnliche Schlussfolgerung dieser Theorie hat
    eine deutlich panpsychistische Prägung:
    [im folgenden ein Zitat von Tononi] „Unter diesem Gesichtspunkt ist das Erleben, also die Bündelung von Informationen, eine fundamentale Eigenschaft, so wie auch Masse, Ladung und Energie welche sind. Daraus folgt, dass jedes physikalische System in dem Maße ein subjektives Erleben hat, wie es in der Lage ist, Informationen zu bündeln – ganz egal, aus was dieses System besteht. […] Meine Theorie behauptet, dass das Bewusstsein keine Allesoder-nichts-Eigenschaft, sondern abgestuft ist. […] Streng genommen impliziert meine Theorie, dass sogar eine binäre Photodiode nicht völlig unbewusst ist, sondern genau 1 Bit Bewusstsein erlebt. […] Im Gegensatz zum traditionellen Panpsychismus spricht meine Theorie jedoch nicht wahllos allen Dingen ein Bewusstsein zu.“

    Im Gegensatz zum traditionellen Panpsychismus geht der Graduelle Panpsychismus von einer gestuften Komplexität des Bewusstseins aus. Damit stimmt er nicht nur mit Tononis Theorie überein, er findet in dieser Theorie sogar eine neurowissenschaftliche Bestätigung des Komplexitäts-Bewusstseins-Gesetzes: Die Komplexität des bewussten Erlebens ist stets an die neurophysiologische (und damit materielle) Beschaffenheit gebunden. Je komplexer diese Beschaffenheit ausfällt, desto komplexer kann auch das Bewusstsein sein.
    Daher hat ein Atom vielleicht nur eine simple binäre Wahrnehmung der Wirklichkeit, indem es Ja-Nein-Informationen verarbeitet, die 1 Bit entsprechen. Die graduelle Steigerung des Geistigen korrespondiert
    mit der jeweiligen materiellen Beschaffenheit eines Dings: So findet sich im Tierreich eine gestufte Anordnung geistiger Fähigkeiten, die beim Menschen schließlich ihren (vorläufigen) Höhepunkt
    findet.”

  111. @Patrick Spät: Danke für 10 Tage!

    Lieber Patrick,

    die zehn “zugesagten” Online-Tage sind rum, was natürlich nicht bedeutet, dass die Blogdiskussion enden muss – aber eben, dass Du Dein Versprechen eingehalten hast, uns zehn Tage lang nach Deinem Web-Interview zur Verfügung zu stehen.

    Ich freue mich, mitteilen zu können, dass mit bislang 118 Kommentaren und über 3.300 Zugriffen dieser Blogpost eine sehr rege Nachfrage gefunden hat und sicher auch weiter finden wird. Dass Philosophie uns anregen kann, Begriffe und Konzepte immer mal wieder neu zu beleuchten hat sich m.E. eindrucksvoll gezeigt: Ich habe mir u.a. “Selbstorganisation” und “Emergenz” für kommende Vertiefungen vorgenommen.

    Dir von mir ein herzliches Danke für Deine Bereitschaft zur Antwort & Diskussion, Deinem philosophischen Streben & Buch weiterhin viel Erfolg!

    Michael

  112. @ Michael, @ alle Kommentatoren

    @ Michael:
    Tausend Dank für Deine Worte – und für die Einladung!

    @ alle Kommentatoren:
    Vielen Dank für die zahlreichen Kommentare, für die Blumen und Dornen und für manch kritischen Einwand, der das Weltbild des Panpsychismus hoffentlich bereichern wird.

  113. Sehr geehrter Herr Spät!

    Diese email erreichte Sie aus Peru.
    Ich würde Ihnen gerne eine email mit Anhang! senden, über sehr intensive eigene Erfahrungen und Handlungen!! in der “Sphäre der Ideen”.
    Bitte lassen Sie mir Ihre email Adresse zukommen.

    mit freundlichem Gruß
    U. Mattheyer

  114. Pingback:Quanten, Physik & Philosophie – Was ist eigentlich Realität? – Einführung in die Tropenontologie von Meinard Kuhlmann › Natur des Glaubens › SciLogs - Wissenschaftsblogs

Schreibe einen Kommentar